Anda di halaman 1dari 83

Pilihlah salah satu jawaban yang paling tepat! 3.

1. Perhatikan kutipan teks pidato berikut!


Yang sangat terhormat Bapak Kepala SMP Darul Fikri, Bapak Akmal
Fauzan. Pertama, marilah kita bersyukur ke hadirat Tuhan yang
Maha Pengasih.
Perbaikan sapaan pada kutipan teks pidato tersebut adalah ...
A. Yang terhormat Bapak Kepala SMP Darul Fikri, Bapak Akmal
Fauzan
B. Yang terhormat Kepala SMP Darul Fikri, Bapak Akmal Fauzan Agar menjadi padu, perbaikan kalimat nomor (3) yang tepat adalah…
C. Yang terhormat Bapak Kepala SMP Darul Fikri, Akmal Fauzan A. Di Pakistan dan dunia akan berkabung atas kepergian seorang
D. Yang sangat hormati Kepala SMP Darul Fikri. perempuan yang memberikan banyak inspirasi bagi perjuangan
kaumnya.
2. . B. Pakistan dan dunia berkabung atas kepergian seorang perempuan
Penggunaan tekhnologi penangkapan ikan di laut
yang memberikan banyak inspirasi bagi perjuangan kaumnya.
hendaknya diwaspadai agar nggak sampai mematikan mata
C. Pakistan dan dunia akan merayakan kepergian seorang
pencaharian nelayan tradisionil, mengingat sistim
perempuan yang memberikan banyak inspirasi bagi perjuangan
penangkapan ikan pada zaman sekarang lebih modern.
Kata baku dari kata-kata bercetak miring seharusnya adalah ...
kaumnya.
A. tecnologi, tak, tradisionil, sistem
D.Negara Pakistan dan dunia diperingati kepergian seorang
(1) Dunia sungguh dibuat terkejut dan terguncang oleh tragedi
B. technologi, nggak, traditional, sistim
perempuan yang memberikan banyak inspirasi bagi perjuangan
pembunuhan pemimpin oposisi Pakistan, mantan Perdana
C. technologi, tiada, tradisional, sistem
kaumnya.
Menteri Benazir Butho. (2) Benazir tewas ditembak hari Kamis,
D.teknologi, tidak, tradisional, sistem
4. (1) Sejak peristiwa itu hingga kini aku tak pernah lagi bertemu
27 Desember 2007 oleh penyerang bom bunuh diri. (3) Pakistan
dengannya. (2) Terhadap dia sama sekali aku tak punya pikiran
dan dunia akan merayakan atas kepergian seorang perempuan
apa-apa. (3) Sudah sangat lama sekali bagiku peristiwa itu sudah
yang memberikan banyak inspirasi bagi perjuangan perempuan.
kulupakan. (4) Lebih baik, aku memikirkan masa depanku.
(4) Beliau menjadi pejuang demokrasi dan modernisasi bagi
negaranya.
Penulisan kalimat yang kurang efektif pada paragraf di atas 7. BANDUNG, (PR).- Sekolah Asih Putera Cimahi tampil sebagai
terdapat pada kalimat …. pemenang umum setelah mampu memboyong semua sebutan juara
A. kesatu C. kedua dari tingkat TK, SD, SMP, dan SMA pada turnamen futsal Al Irsyad
B. ketiga D. keempat Satya Cup 2017 seBandung Raya di Lapangan Olah Raga Al Irsyad
Satya Islamic School Kota Baru Parahyangan, Padalarang, yang
5. Perhatikan kalimat berikut! berakhir Senin (17/09).
Kami selalu sarapan pagi agar supaya tidak sakit perut ketika Agar menjadi paragraf yang efektif, penggunaan kata-kata yang
berolahraga. bercetak miring seharusnya …
Perbaikan kalimat tidak efektif tersebut yang tepat adalah … A. juara, gelar, se-Bandungraya, olah raga
A. Kami selalu sarapan pagi agar tidak sakit perut ketika B. juara, gelar, se-Bandung Raya, olah raga
C. juara, gelar, se-Bandungraya, olahraga
berolahraga.
D.juara, gelar, se-Bandung Raya, olahraga
B. Kami selalu sarapan pagi supaya tidak sakit perut ketika
berolahraga.
C. ketika berolahraga kami selalu sarapan agar supaya tidak sakit
8. (1) Dampak yang dihasilkan oleh sampah sangatlah banyak. (2)
perut.
sampah dapat menyebabkan lingkungan menjadi kotor. (3) Selain
D. Kami selalu sarapan agar tidak sakit perut ketika berolahraga.
itu, sampah dapat menjadi sumber penyakit. (4) Kita menginginkan
kesejahteraan. (5) oleh karena itu, sampah seharusnya diolah
6. (1) Lingkungan adalah segala sesuatu yang ada di sekitar manusia. menjadi kompos.
(2) Lingkungan bermacam-macam, di antaranya lingkungan rumah Perbaikan paragaraf yang tidak padu di atas adalah ,…
dan lingkungan sekolah. (3) Lingkungan tersebut harus dirawat dan A. Menghilangkan kalimat ke (4)
dijaga kebersihannya. (4) Jika tidak dirawat dan dijaga, lingkungan B. Menghilangkan kalimat ke (5)
akan kotor sehingga menjadi sumber berbagai penyakit. (5) setiap C. Mengganti diolah dengan dikemas
hari Minggu diadakan kerja bakti. D. Mengganti selain itu dengan kecuali itu
Kalimat tidak padu pada teks tersebut adalah … Diki mengikuti lomba Cipta Puisi dan
A. (2) C. (4) Cipta Cerpen. Mendampingi (1) sekolahnya
B. (3) D. (5) 9. pada FLS2N
Bacalah tahunberikut!
paragraf 2015. FLS2N, pernah
(2) dilakukan setiap tahun oleh
Kementerian Pendidikan dan Kebudayaan.
Hanya tekad dan keberanian cikal (3)
utamanya.
tanaman yang dipupuk dengan pupuk kandang lebih banyak.
Tanaman pun terlihat lebih kuat dan sehat. Tanaman yang
dipupuk dengan pupuk kimia memiliki daun dan bunga yang lebih
sedikit. Selain itu, pupuk kandang mampu menjaga unsur
keseimbangan unsur hara dalam tanah sehingga tanaman tetap
Perbaikan kata bercetak miring pada paragraph tersebut adalah … mendapatkan gizi secara seimbang. Berbeda dengan pupuk kimia
A. (1) mengirim;(2) kadang-kadang; (3) niat yang bisa membuat tanah jadi rusak.
B. (1) mewakili; (2) rutin; (3) modal Perbaikan istilah gizi yang tepat pada teks di atas adalah …
C. (1) mengutus; (2) biasanya;(3) sasaran A. nutrisi C. asupan
D. (1) membawa; (2) jarang; (3) pokok B. vitamin D. kalori
13. Grup pemberantasan korupsi akan memeriksa pejabat yang terduga
10. Berdasarkan hasil … dari berbagai sumber menunjukkan bahwa menerima suap. Pemeriksaan ini rencananya akan diperiksakan di
siswa yang sering menyontek adalah siswa yang jarang masuk gedung KPK, Jakarta.
sekolah. Perbaikan kata bercetak miring yang tepat adalah …
Kata yang tepat untuk melengkapi kalimat tersebut adalah … A. Kelompok, menduga, dilakukan
A. penelitian C. perbandingan B. Kelompok, menduga, digelar
B. pertemuan D. perkataan C. Tim, diduga, dilakukan
D.Tim, diduga, diperkirakan
11. Team favoritnya tereliminasi pada babak semifinal.
Penulisan kata yang salah pada kalimat tersebut adalah … 14. Setelah mendapat penjelasan dari pembina OSIS, akhirnya mereka
A. team C. favoritnya […] untuk saling memaafkan.
B. tereliminasi D. semifinal Kata yang tepat untuk melengkapi kalimat di atas adalah …
A. Bersalaman-salaman
12. Tanaman yang dipupuk dengan pupuk kandang ternyata lebih B. Salam-salaman
subur dan tahan terhadap penyakit. Daun dan bunga pada C. Bersalam-salam
D. Bersalaman bertujuan untuk menambahkan ketakwaan terhadap Tuhan Yang
Maha Esa.
15. Rumah itu sudah lama ditinggalkan pemiliknya. Bahkan, Perbaikan penggunaan kata memeroleh dan menambahkan yang
beberapa hari yang lalu dipasang plang akan dijual. Namun, tepat adalah …
rumah besar dan strategik seperti itu tentu berharga sangat A. mencari, menumbuhkan
mahal. Hanya orang iseng yang mencoba menghubungi nomor B. mencapai, meningkatkan
telepon di plang itu. C. mendapatkan, menstabilkan
Kata dengan ejaan yang tidak tepat dan perbaikannya pada teks D.menggapai, mengembangkan
tersebut adalah … 18. Banyak buku-buku di perpustakaan itu belum dibaca.
tertulis Perbaikan Penggunaan kata banyak pada kalimat tersebut salah karena ….
A. dijual di jual A. Kata ulang buku-buku sudah bermakna banyak.
B. strategik Strategis B. Kalimat tidak dapat didahului oleh kata bilangan.
C. plang Pelang C. Kata banyak tidak menunjukkan jumlah yang pasti.
D. nomor Nomer D.Kata banyak tidak dapat diletakkan di awal kalimat.

16. Bantuan kepada pengungsi yang dalam bentuk daripada bahan 19. Sistem pekerja Outsourcing yang kini diterapkan banyak
makanan sangat dianjurkan. perusahaan dinilai menyengsarakan dan merugikan pekerja.
Perbaikan kalimat tersebut adalah menghilangkan kata …. Penulisan kata outsourcing pada kalimat tersebut salah karena …
A. yang, daripada A. Kata outsourcing tidak dibenarkan ditulis kapital karena bukan
B. bahan, daripada kata serapan.
C. dalam, sangat B. Kata outsourcing tidak dibenarkan ditulis kapital karena sudah
D.bentuk, makanan ditulis miring.
C. Kata outsourcing tidak dibenarkan ditulis kapital karena bukan
17. Pelajaran Bahasa Indonesia penting dalam rangka memeroleh nama orang.
tujuan pendidikan nasional. Pendidikan berdasarkan Pancasila
D.Kata outsourcing tidak dibenarkan ditulis kapital karena belum
ada padanannya.

20. Anak yang terampil […] sepeda gunung itu menjadi juara dalam
lomba sepeda gunung antarsekolah.
Kata yang tepat untuk melengkapi kalimat tersebut adalah …
A. memakai C. mengendarai
B. membawa D. menggunakan 23. Cermatilah kalimat berikut!
21. [….] Pagi itu aku ditunjuk mengikuti lomba menyanyikan lagu Di toko Megaphone dijual tiga jenis ponsel Samsung Oppo dan
wajib dalam rangka memperingati Hari Pendidikan Nasional. Aku Penggunaan
Black Berry tanda baca yang tepat pada kalimat di atas adalah …
A. Di toko Megaphone dijual tiga jenis ponsel; Samsung, Oppo dan
mendapat juara pertama dalam lomba tersebut.
Black Berry.
Kalimat yang tepat untuk melengkapi teks tersebut adalah …
B. Di toko Megaphone dijual tiga jenis ponsel: Samsung, Oppo, dan
A. Hari itu aku sedih dan bahagia.
Black Berry.
B. Hari itu aku bangga dan terharu. C. Di toko Megaphone dijual tiga jenis ponsel, Samsung, Oppo, dan
C. Hari itu aku mneyesal mengapa aku ditunjuk. Black Berry.
D.Di toko Megaphone dijual tiga, jenis ponsel Samsung, Oppo, dan
D.Hari itu aku menyesal mengapa aku tidak dapat juara.
Black Berry.

22. Sudah dua Minggu ayah di Malang untuk meneliti apel Malang.
24. Penulisan tanda hubung yang tidak tepat terdapat dalam kalimat ...
Perbaikan penulisan huruf kapital pada kalimat tersebut yang tepat
A. Dalam HUT RI yang ke-70 akan diadakan perlombaan
adalah...
kebersihan antarkelas.
A. Sudah dua Minggu ayah di Malang untuk meneliti Apel malang.
B. Pembicaraannya masih tampak ke-Jawa-jawa-an.
B. Sudah dua minggu Ayah di Malang untuk meneliti apel Malang.
C. Gedung ini dibangun kira-kira tahun 90-an.
C. Sudah dua Minggu Ayah di Malang untuk meneliti Apel Malang
D.Ayahnya telah lama di-PHK perusahaan ini.
D.Sudah dua minggu ayah di Malang untuk meneliti apel malang.
25. Tim futsal MTs Asih Putera pada bulan Pebruari akan melaksanakan
pertandingan persahabatan.
Kesalahan ejaan pada kalimat di atas adalah ….
A. tim C. futsal 28. Surat itu dikirim oleh Naufal Rizky, SH a/n PT Megah Abadi.
B. Pebruari D. pertandingan Perbaikan tanda baca pada kalimat tersebut adalah …
A. Surat itu dikirim oleh Naufal Rizky, S.H. a.n. PT Megah Abadi.
26. Situs kerajaan Majapahit terletak di kecamatan Trowulan, B. Surat itu dikirim oleh Naufal Rizky, SH a.n. PT Megah Abadi.
kabupaten Mojokerto. C. Surat itu dikirim oleh Naufa; Rizky, SH a/n. PT Megah Abadi.
Penulisan huruf kapital yang tepat pada kalimat tersebut adalah … D.Surat itu dikirim oleh Naufal Rizky, S.H. a.n. P.T. Megah Abadi.
A. Situs Kerajaan Majapahit terletak di Kecamatan Trowulan,
Kabupaten Mojokerto. 29. Pembina OSIS akan memimpin rapat kerja program tahunan tetapi
B. Situs kerajaan Majapahit terletak di Kecamatan Trowulan, tiba-tiba ia ditugasi oleh Kepala Sekolah untuk rapat di Dinas
Kabupaten Mojokerto. Pendidikan Kota mewakili beliau.
C. Situs kerajaan Majapahit terletak di Kecamatan Trowulan, Penggunaan tanda baca yang tepat pada kalimat tersebut adalah
kabupaten Mojokerto. mencantunkan …
D. Situs kerajaan majapahit terletak di kecamatan trowulan, A. tanda koma (,) setelah singkatan OSIS
kabupaten mojokerto. B. tanda koma (,) setelah kata tahunan
C. tanda titik koma (;) setelah singkatan OSIS
27. Orang utan biasanya mengonsumsi: buah-buahan, biji-bijian, dan D.tanda titik koma (;) setelah kata tetapi
daun-daunan.
Penggunaan tanda baca yang tidak tepat pada kalimat tersebut
adalah …
A. titik (.) C. koma (,)
B. hubung (-) D. titik dua (:)
30. Irene
Irene Sukandar lahir di Jakarta pada tanggal 7 April 1992. Dia
adalah Grand Master Catur Putri Indonesia. Dia telah meraih
berbagai macam gelar, baik tingkat nasional maupun
internasional. Gelar yang berhasil diraihnya seperti : Master
Percasi (MP), Master Nasional Wanita (MNW) dan Master Fide
Wanita (MFW).

Perbaikan penulisan tanda baca pada kalimat terakhir yang tepat


adalah …
A. Gelar yang berhasil diraihnya seperti: Master Percasi (MP),
Master Nasional Wanita (MNW), dan Master Fide Wanita (MFW).
B. Gelar yang berhasil diraihnya seperti Master Percasi (MP), Master
Nasional Wanita (MNW), dan Master Fide Wanita (MFW).
C. Gelar yang berhasil diraihnya, seperti: Master Percasi (MP),
Master Nasional Wanita (MNW) dan Master Fide Wanita (MFW).
D.Gelar yang berhasil diraihnya, seperti Master Percasi (MP),
Master Nasional Wanita (MNW) dan Master Fide Wanita (MFW).
PEMANTAPAN 2

~MAKNA~
Pilihlah salah satu jawaban yang paling tepat! Arti istilah fatwa dalam paragraf tersebut adalah ....
1. [….] digital di depan ruang dingin vaksin menunjukkan angka 3 A. keputusan D. imbauan
derajat celcius. Angka tersebut tergolong normal bagi ratusan ribu B. usulan C. larangan
vaksin di dalamnya Vaksin-vaksin tergolong sensitive beku sehingga
suhu di dalam cold room haus stabil antara 3-8 derajat celcius. 4. Dalam peninjauan realisasi PNPM Mandiri Provinsi Jawa Barat di
Istilah yang tepat untuk melengkapi teks tersebut adalah …
Kota Bogor, beberapa waktu yang lalu, Presiden Susilo Bambang
A. Inkubator
B. Proyektor Yudhoyono mengungkapkan keprihatinannya atas iklan politik yang
C. Stetoskop
mengangkat tema angka kemiskinan. Menurut Presiden, iklan
D. Termometer
tersebut tidak proporsional dan cenderung mendiskreditkan
2. Bu Iren sudah memasukan nama Zahra sebagai peserta lomba
Pemerintah.
baca puisi. Makna istilah mendiskreditkan pada teks di atas adalah ….
Makna kata memasukkan pada kalimat tersebut adalah … A. perbedaan perlakukan yang menyimpang tentang kondisi
A. Membicarakan
kemiskinan
B. Mengusulkan
B. berusaha untuk menjelekkan atau memperlemah kewibawaan
C. Membawakan
D. Mendaftarkan pemerintah
C. ketidaksesuaikan data angka kemiskinan dengan keadaan yang
3. Terkesan aneh bin ajaib jika benar Pemerintah melalui
sebenarnya
Kementerian Energi dan Sumber Daya Mineral (ESDM) dengan D.ketidakpercayaan terhadap data kemiskinan yang disampaikan
menggandeng Majelis Ulama Indonesia (MUI) mengeluarkan fatwa pemerintah
5. Satu minggu Tono berkenalan dengan Tini, wanita yang kini
haram menggunakan BBM bersubsidi bagi orang mampu. Langkah
hendak dinikahinya. Akan tetapi, takdir harus memisahkan
demikian merefleksikan ketidakberdayaan Pemerintah merumuskan
keduanya. Tini meninggal dunia akibat tertimpa pohon saat hujan
kebijakan mengatasi subsidi bahan bakar minyak yang dinilai tidak
lebat. Padahal mereka baru belajar mengenal satu dengan yang
tepat sasaran. Kekacauan kebijakan subsidi terletak pada desain
lainnya. Boleh diibaratkan cintanya seperti embun diujung rambut.
kebijakan itu sendiri tanpa dikaitkan dengan problem nasional lain
Makna kalimat seperti embun diujung rambut adalah ....
terkait energi. Semua ini sebenarnya disebabkan oleh ketidakjelasan
A. cintanya belum mendalam
Pemerintah dalam mengelola energi nasional.
B. cintanya tak diragukan
C. kasih sayangnya begitu mendalam B. Jeruk nipis adalah buah yang berwarna hijau berbentuk bulat
D. cintanya seperti cinta monyet kecil seperti bola tetapi memiliki khasiat yang tidak kecil seperti
bentuknya.
6. Pertumbuhan industri penerbangan nasional akhir-akhir ini
C. Jeruk nipis biasanya digunakan untuk campuran minuman dan
menunjukkan peningkatan. Tentu saja hal ini memberikan dampak
berkhasiat menyembuhkan penyakit.
positif terhadap perekonomian. Masyarakat mendapatkan D.Jeruk nipis tanpa biji telah menembus pasar mancanegara.
keuntungan terutama dari sisi harga perjalanan yang semakin
murah dan mudah. Namun keselamatan penerbangan tetap harus 8. Selain melakukan diet, mengolah tubuh merupakan cara yang
diperhatikan oleh relugator dan operator penerbangan. dapat ditempuh untuk menyiasati masalah kegemukan. Akan tetapi,
Makna istilah relugator dan operator dalam paragraph tersebut hal ini pada dasarnya tidak hanya berlaku untuk anak yang
adalah … mengalami kegemukan, melainkan kepada setiap anak. Hanya saja,
A. Pembuat aturan – petugas yang menjalankan
beberapa anak merasa malas untuk melakukan kegiatan fisik,
B. Mesin pengatur – petugas pengatur
terlebih jika layar kaca telah menghipnotisnya.
C. Mekanik – petugas penerbangan
Makna tersurat paragraf di atas adalah ....
D. Pelaksana aturan – petugas operasi
A. Setiap anak sebaiknya tidak terlalu terhipnotis dengan televisi.
B. Mengolah tubuh sangat diperlukan untuk semua anak tidak
7. Perhatikan teks berikut ! hanya untuk anak mengalami masalah kegemukan.
C. Layar kaca terlalu menghipnotis anak dan membuat anak malas
Jeruk nipis mulai populer sebagai salah satu jenis buah tropis yang
untuk melakukan diet
mempunyai nilai ekonomi tinggi. Pasaran buah jeruk nipis telah
D. Diet adalah salah satu cara untuk mengatasi kegemukan.
menembus perdagangan antarnegara, termasuk jeruk nipis tanpa
9. 1) Penelitian Yayasan Lembaga Konsumen Indonesia (YLKI)
biji. Beberapa petani di berbagai daerah telah beranjak
terhadap Air Minum Dalam Kemasan (AMDK) yang diproduksi oleh
mengebunkan jeruk nipis dalam skala usaha lebih luas dan intensif.
21 perusahaan menemukan AMDK gelas yang tidak layak minum.
Bagi peminat bisnis tani jeruk nipis yang tidak memiliki lahan cukup
(2) Selain itu, ada yang layak minum, tetapi dengan catatan habis
luas, dapat mempertimbangkan penanaman dalam pot.
dicermati lagi proses produksinya. (3) Jika tidak, ia bisa menjadi
Makna tersurat dari paragraf di atas adalah ....
tak layak minum. (4) Penelitian YLKI ini dilatarbelakangi oleh
A. Jeruk nipis sangat bermanfaat untuk menghilangkan bau amis.
ditemukannya AMDK gelas yang tercemar karena bocor dan 11. Kapolri Jenderal Pol Badrodin Haiti menyambut positif
ditemukannya nomor register yang berbeda-beda dalam satu dus. penghargaan kepada orang-orang yang telah bekerja keras tanpa
(5) Bahkan, ada AMDK yang kandungan koloninya jauh melebihi pamrih, memberikan dampak positif bagi masyarakat Indonesia
SNI sementara tanggal kedaluwarsa AMDK tersebut masih satu melalui program Pahlawan untuk Indonesia tahun 2015. Badrodin
tahun lebih. pun memberikan apresiasi kepada salah satu peraih penghargaan
Makna tersurat paragraf di atas adalah .... yang juga merupakan Anggota Polwan dari Polres Nganjuk, Jawa
A. Standar SNI untuk AMDK gelas boleh melebihi asal batas waktu Timur, yakni Aiptu Naniek Yuliati. Nanik mendapat penghargaan
kedaluwarsanya masih lama. dalam kategori pelayanan publik karena mampu melayani dan
B. Terdapat AMDK gelas yang tidak layak minum berdasarkan
menangani puluhan warga Nganjuk yang dipasung lantaran
penelitian YLKI.
gangguan jiwa. (sindonews.com)
C. Kandungan koloni pada AMDK gelas tidak akan berubah sejak
Kata apresiasi yang merujuk pada bacaan di atas bermakna....
dari pabrik hingga konsumen.
D. YLKI hanya bertugas meneliti AMDK gelas dan tidak perlu A. pembuktian terhadap sesuatu
meneliti AMDK botol atau galon. B. penghargaan terhadap sesuatu
C. petunjuk terhadap sesuatu
10. Grup mahasiswa kedokteran akan melakukan ujicoba di posko-
D. pengamanan terhadap sesuatu
posko bencana banjir yang tersebar di beberapa RW. Mereka juga
membawa obat-obatan yang disumbangkan oleh pemilik apotek
12. 1) Kekerasan terhadap anak semakin marak. 2) Satgas
terhadap menderitanya rakyat yang terkena musibah.
Perlindungan Anak, misalnya, setiap hari menerima enam laporan
Perbaikan pilihan kata bercetak miring yang tepat adalah ...
kekerasan dan kejahatan terhadap anak. 3) Laporan yang sama
A. tim, praktik, penderita
banyaknya diterima juga di daerah oleh lembaga yang bergerak
B. tim, tugas, penderitaan
dalam perlindungan anak. 4) Tingginya kekerasan terhadap anak
C. kelompok, praktik, penderitaan
memperlihatkan ketidakpedulian kita terhadap hak anak untuk
D.kelompok, tugas, menderita
tumbuh dan berkembang dengan wajar, bebas dari rasa takut, dan
ancaman. 5) Oleh karena itu, kita menuntut Kementerian PPPA
membuat kebijakan nasional yang lebih konkret dalam melindungi
anak. (kompas.com) 14. Pada umumnya kucing peliharaan tidak berbahaya bagi manusia
Mengapa kekerasan terhadap anak masih terus terjadi? karena tubuhnya yang kecil. Namun, tidak menutup kemungkinan
A. Kementerian Perlindungan Perempuan dan Anak belum hewan ini dapat menularkan penyakit. Penyakit yang mungkin
membijak kebijakan nasional. ditularkan di antaranya rabies akibat gigitannya dan gangguan
B. Satgas Perlindungan Anak hanya menerima laporan kekerasan pernapasan oleh bulu-bulunya.
terhadap anak tanpa bertindak. Informasi tersurat yang terdapat dalam kutipan teks tersebut
C. Pemberitaan tentang kekerasan terhadap anak kurang disorot adalah …
oleh media massa. A. Kucing merupakan hewan yang banyak dipelihara manusia
D.Ketidakpedulian lita dan masyarakat terhadap pertumbuhan dan B. Penyakit yang dapa ditularkan kucing di antaranya rabies
perkembangan anak. C. Tubuh kucing yang kecil membuat hewan ini disukai banyak
orang
13. Selama ini, masyarakat mengenal sosok Sri Rossa Roslaina D.Kucing dapat menularkan gangguan pernapasan oleh cakarnya
Handiyani alias Rossa sebagai penyanyi. Namun, siapa sangka
ibunda dari Rizky Langit Ramadhan ini jago berbisnis. Rossa 15. “Mila, minggu depan kita harus pindah ke Yogyakarta. Ayah
memilih investasi pada bidang property. Ia menekuni bisnis jual dipindahtugakan di sana.” Bagaikan petir di siang bolong
beli rumah sejak 2002. Guna melipatgandakan asetnya, Rossa menyambar Mila, yang seketika itu langsung diam mematung.
membeli rumah di pasar sekunder untuk investasi. Lalu, penyanyi Makna simbol petir di siang bolong dalam kutipan ceirta tersebut
bersuara sopran ini akan menjualnya kembali. adalah …
Informasi tersurat yang terdapat pada kutipan teks tersebut A. Merasa sedih
adalah… B. Merasa bingung
A. Rossa sekarang beralih profesi menjadi penjual rumah C. Sangat terkejut
B. Selain, jago menyanyi, Rossa joga jago bisnis D. Sangat heran
C. Rossa mempunyai suara yang merdu
D. Rossa seorang penyanyi yang mempunyai anak tunggal
16. Nama saya Kalinda Sachi Elmeera, tetapi cukup dipanggil Sachi pembicaraan diberbagai kesempatan dan pemberitaan media
saja. massa.
Makna kata cukup dalam teks di atas adalah … Ungkapan yang tepat untuk ilustrasi di atas adalah …
A. Lengkap sekali A. buah tangan C. buah pikiran
B. Hanya lumayan
C. Tidak beruntung
PEMANTAPAN 3
B. buah bibir D. buah pena

D. Sudah memadai

17. Pada suatu hari Tio dan Rizky berkelahi hanya karena hal sepele.
Mereka saling menonjok dan meninju. Hingga akhirnya, Naufal
dan teman-teman 9B melerainya. Rizky terlihat babak belur,
sementara Tio masih Nampak memendam kemarahannya. Mereka
pun dibawa ke ruang guru untuk menyelesaikan
permasalahannya.
Peribahasa yang tepat untuk menggambarkan ilustrasi di atas
adalah …
A. Tak ada yang menang dan kalah
B. Kalah jadi abu menang jadi arang
C. Bagai telur di ujung tanduk
D. Bagai mencencang air

18. Setelah melalui pertandingan yang sengit. Tim futsal MTs Asih
Putera berhasil menjadi juara dalam Liga Futsal antar-SMP/MTs
se-Jawa Barat kali ini. Tim futsal MTs Asih Putera menjadi bahan
TEKS BERITA
Pilihlah salah satu jawaban yang paling tepat! B. siapa D. di mana

1. Presiden Gloria Macapagal-Arroyo telah menandatangani Undang- 4. Berdasarkan strukturnya teks berita tersebut termasuk ke dalam …
Undang yang menyatakan bahwa Idul Adha sebagai Hari Raya A. orientasi C. peristiwa
Nasional dan dijadikan hari libur nasional. Demikian seperti B. sumber berita D. reorientasi
diberitakan Inquirer.net edisi Senin (28/8).Libur lebaran haji di
Filipina merupakan UU ke 9.849 yang ditandatangani 11 Desember 5. Berita 1
tahun lalu. Dalam UU itu disebutkan, hari ke-10 Zulhijjah, bulan Pencemaran air dengan skala terberat di badan sungai yang
ke-12 kalender Islam, dinyatakan sebagai hari libur nasional untuk melintasi Jakarta, Surabaya, dan Palembang, semakin meningkat.
peringatan Idul Adha. Idul Adha adalah hari ke-10 dalam bulan haji Penyebab utama tercemarnya air di badan sungai tersebut adalah
saat umat Islam melaksanakan haji untuk mengikuti jejak Nabi limbah industri. Pada tahun 2020, kondisi tersebut diperkirakan
Ibrahim AS. Idul Adha juga dikenal sebagai Hari Raya Kurban. semakin buruk. Hal ini disebabkan karena industry di Indonesia
Topik berita di atas adalah … pada tahun itu akan meningkat 13 kali lipat. Pakar lingkungan
A. Idul Adha adalah bulan Islam ke-10. hidup dari Universitas Sriwijaya, Dr. Hilda Zulkifli, mengemukakan
B. Penandatanganan UU Idul Adha sebagai hari libur nasional. hal itu seperti dikutip Antara, Kamis (9/1).
C. Liburan haji untuk muslim Filipina.
D. Arroyo menyetujui libur Idul Adha
Berita 2
Pakar lingkungan hidup dari Universitas Sriwijaya, Dr. Hilda
2. (1) Empat nelayan asal Cilacap hilang. (2) Empat nelayan itu
Zulkifli mengemukakan bahwa pencemaran air di badan sungai
bernama Hendro (35), Marno (50), Toyo (45), dan Ranto (45). (3)
yang melintasi Jakarta, Surabaya, Medan, dan Palembang, semakin
Kabar itu diketahui pukul 10.00 WIB setelah salah satu nelayan,
mengkhawatirkan. Penyebab utama pencemaran air di badan
Kliwon (40), selamat dari peristiwa itu. (4) Mereka hilang setelah
sungai tersebut adalah limbah industry. Pada tahun 2020, kondisi
kapal yang mereka tumpangi pecah berkeping-keping dihantam
tersebut diperkirakan semakin buruk. Hal ini disebabkan industry
gelombang setinggi tiga meter. (5) Ketua SAR Cilacap berharap
di Indonesia pada tahun itu akan meningkat 13 kali lipat.
korban bisa ditemukan dalam keadaan selamat. (Sindo, 28
Dilihat dari gaya penyajiannya, perbedaan kedua teks tersebut
September 2016)
adalah ,..
Opini berita di atas terdapat dalam kalimat …
Teks I Teks 2
A. kedua C. keempat
A. Menonjolkan waktu Menonjolkan tempat
B. ketiga D. kelima
kejadian kasus kejadian kasus
B. Mengungkapkan akibat Mengungkapkan siapa
3. Unsur berita yang tidak terdapat dalam kutipan berita nomor 6
kasus yang mengakibatkan kasus
adalah …
C. Menjelaskan alas an dan Menjelaskan akibat
A. bagaimana C. mengapa
akibat
D. Menonjolkan kasus apa Menonjolkan siapa yang Sekitar 40 gubuk milik pemulung habis terbakar di kawasan
yang terjadi mengungkapkan kasus Kebagusan, Pasar Minggu, Jakarta Selatan, Selasa (10/9). Api juga
melalap sebagian gedung industry pemolesan marmer yang berimpitan
6. Teks Berita I dengan permukiman pemulung. Kebakaran itu mengakibatkan
Pemerintah Provinsi DKI Jakarta memulai sertifikasi kesehatan puluhan perabot kayu jati senilai ratusan juta rupiah turut habis
unggas secara bersamaan di berbagai lokasi. Dana sebesar Rp 3 terbakar.
miliar dianggarkan untuk sertifikasi itu. Proses sertifikasi unggas Kebakaran itu terjadi sekitar pukul 09.00 ketika sebagian besra
hias ditandai dengan pemeriksaan 11 unggas milik Gubernur DKI di pemulung tengah bekerja memulung hingga ke daerah Ragunan.
rumah dinasnya Hanya sebagian dari perempuan dan anak-anak yang tinggal di gubuk
Teks Berita 2 ketika kebakaran terjadi. Kebakaran tersebut diduga disebabkan oleh
Setelah burung dan unggas milik Gubernur Sutiyoso, hari ini giliran hubungan pendek arus listrik.
burung dan unggas para menteri di kompleks Widya Chandra akan “Padahal, besok saya akan pulang kampong ke Indramayu. Barang
dilakukan pemeriksaan kesehatan untuk mendapat sertifikat sehat. sudah disiapkan. Malah uang tabungan ikut habis terbakar,” kata
Petugas akan mendatangi kediaman Aburizal. Burung yang akan Dasin (30), yang tabungannya sebesar Rp1,5 juta habis terbakar.
diperiksa adalah burung nuri kepala hitam
Teks 2
Perbedaan penyajian kedua teks tersebut adalah …
Kebakaran di Tanah Sereal
Teks I Teks 2 Kebakaran juga melanda permukiman padat di Kelurahan Sereal,
A. siapa-berapa- kapan-bagaimana-apa Kecamatan Tambora, Jakarta Barat, Selasa siang. Petugas piket Suku
bagaimana Dinas Kebakaran Jakarta Barat, Sarino, menjelaskan bahwa
B. siapa-di mana-berapa kapan-di mana-siapa kebakaran dilaporkan pukul 12.40. Dikerahkan 24 mobil pemadam
kebakaran dari Jakarta Barat, Jakarta Pusat, dan Kantor Dinas.
“Ada 45 keluarga yang menjadi korban kebakaran. tidak ada yang
C. apa-berapa-kapan mengapa-bagaimana- meninggal dalam musibah kebakaran tersebut.” kata Sarino. Api
siapa padam pukul 14.30. Kebakaran kerap terjadi di Kecamatan Tambora
yang memiliki penduduk terpadat di Asia.
D. apa-kapan-berapa mengapa-apa-siapa

7. Kesamaan informasi dari kedua teks di atas …


Bacalah tek 1 dan 2 untuk menjawab soal no 12-14! A. kebakaran melanda permukiman.
Teks 1 B. kebakaran melanda gubuk-gubuk pemulung.
Gubuk Pemulung Hangus C. penyebab terjadinya arus pendek listrik.
D. Tempat terjadinya di Jakarta Selatan.
Berita 2
8. Pernyataan yang tepat dari kedua teks di atas adalah … Tim Gegana Polda Metro Jaya telah meledakkan satu dari dua
A. Teks berita 1 disajikan secara langsung teks berita 2 paket mencurigakan di Pondok Indah, Jakarta Selatan. Namun, paket
menggunakan gaya cerita. yang diledakkan tersebut tidak mengandung unsur bahan peledak.
B. Teks 1 merupakan perpaduan fakta dan opini, sedangkan teks 2
"Tadi sudah diledakkan satu, tapi tidak ada unsur bom-nya," kata
lebih dominan kepada fakta.
Kapolres Jaksel Gatot Kombes Pol Gatot Eddi, saat dihubungi
C. Teks 1 dan 2 berisi informasi yang sama, yaitu kebakaran yang
detikcom, Jumat (18/3/2011).
terjadi di Jakarta.
D. Teks 2 merupakan perpaduan fakta dan sastra, sedangkan teks 2 Menurut Gatot, kedua buah paket tersebut diterima oleh Indra,
lebih dominan pada sastra. penghuni rumah di Jl Kencana Permai VII, pada hari Senin (14/3).
Paket itu ditujukan kepada Aditya, anak Indra, seorang pegawai
9. Perbedaan penyajian informasi antara kedua teks di atas adalah … Citibank yang sedang bertugas di Shanghai. Namun, paket
A. Kedua teks mengambil informasi dari narasumber yang berbeda, mencurigakan tersebut baru dilaporkan ke polisi pagi ini setelah
yaitu seorang saksi mata kejadian dan pejabat yang berwenang. disimpan beberapa hari. Keluarga memanggil polisi menyusul
B. kedua teks mengambil infromasi dari narasumber yang berbeda, maraknya bom buku belakangan ini. "Iya, pagi inidilaporkan,"katanya.
yaitu orang terkena musibahdan pejabat yang berwenang.
C. waktu kejadian berbeda, yaitu peristiwa dalam teks 1 terjadi pada 10. Persamaan kedua teks berita di atas adalah …
siang hari, sedangkan pada teks 2 terjadi pada malam hari. A. Bom di Jakarta Selatan.
D. penyebab kejadian itu berbeda, teks 1 karena arus pendek, teks 2
B. Teror paket bom mencurigakan
karena kompor meledak.
C. Tim gegana berhasil menjinakkan bom
D.Paket bom yang diledakkan.
Berita 1
Paket yang mencurigakan yang ditemukan di perumahan elite
Monaco, Kota Wisata, Cibubur, Bogor, benar berisi bom dan meledak
sesaat sebelum dijinakkan. Paket bom tersebut bertuliskan 'Isi: Bom
Allahu Akbar'. Yogi mengatakan, paket bom tersebut ditemukan pada
pukul 09.30 WIB oleh seorang petugas kebersihan Kota Wisata di
11. Perbedaan penyajian kedua teks berita di atas adalah …
pinggiran Jl Raya Boulevard, Kota Wisata, Cibubur, Bogor. Pukul 10.00
Teks berita 1 Teks berita 2
WIB, paket mencurigakan tersebut dilaporkan ke Polsek Gunung Putri A. di mana-apa-siapa – kapan siapa – berapa – di mana –
dan Gegana. "Kemudian Gegana melakukan pengecekan awal setelah kapan
diidentifikasi ternyata mengandung logam," jelasnya. Setelah dicek B. apa – kapan – bagaimana – siapa – di mana – kapan –
positif bom, paket bom tersebut dibawa ke suatu lahan kosong di siapa siapa
Kampung China untuk didisposal.
C. di mana – bagaimana –siapa- di mana – berapa – kapan – Pukul 02.05 WIB kemarin, Seorang laki-laki nekat
kapan siapa sebuah Anjungan Tunai Mandiri membakar Anjungan Tunai
(ATM) dibakar. Pelaku tidak Mandiri (ATM). Kejadian itu
D. siapa- apa – kapan – di kapan – siapa – di mana – dikenali. Akibat aksi tersebut, tepat pukul 02.00 WIB dini
mana mengapa gerai ATM meledak hingga hari. Akibatnya gerai ATM
terpental sejauh 10 meter. meledak namun tidak
12. Bandung (PR),- Penanganan dan pemberantasan geng motor menimbukan korban jiwa.
bukan hanya tanggung jawab polisi, melainkan semua kalangan,
termasuk sekolah dan keluarga. Keluarga dan sekolah bertugas A. Teks 1 diawali dengan kapan; teks 2 diawali siapa
memberikan bekal hukum kepada anak-anaknya, sedangkan B. Teks 1 diawali dengan siapa; teks 2 diawali apa
C. Teks 1 diawali dengan apa; teks 2 diawali bagaimana
masalah penegakan hukum menjadi tanggung jawab aparat.
D. Teks 1 diawali dengan bagaimana; teks 2 diawali mengapa
Topik yang terdapat dalam berita di atas adalah ….
A. penanganan dan pemberantasan geng motor tanggung
jawab bersama
B. penegakan hukum geng motor tanggung jawab aparat
C. penanganan geng motor tanggung jawab sekolah dan
keluarganya
D. keluarga dan sekolah bertugas memberikan bekal hidup

13. Penyusunan berita pada tersebut menggunakan struktur ..


A. Balok tegak
B. Piramida terbalik
C. Piramida tegak
D. Segitiga terbalik

14. Perbedaan cara penyajian kedua teks berrita tersebut adalah …

Teks berita 1 Teks berita 2


PEMANTAPAN 4

Teks Drama
Pilihlah salah satu jawaban yang paling tepat!

1. Narasi awal dalam sebuah pementasan drama disebut …


A. epilog
B. dialog
C. monolog
D. prolog

2. Berikut yang tidak termasuk unsur intrinsik drama adalah …


A. petunjuk teknis
B. latar
C. sudut pandang
D. penokohan
Andri : Pokoknya, dia harus saya tegasi. Apapun yang akan terjadi
3. Andi : Selamat malam, Sus ! saya sudah siap!
Suster : Malam Pak Andi. Indra : Jangan begitu, Ndri. Kita harus nyaman dan damai bersama
Andi : Tugas malam, Suster ? kawan. Bukankah kita ini bulan depan harus ujian nasional,
Suster : Betul Pak, saya menggantikan Suster Ani yang sedang penentu masa depan kita?
sakit. Andri : stttt, Bu Ira datang, ayo kita masuk kelas. Sebentar lagi
Dalam penggalan dialog di atas mempunyai latar tempat di … pelajaran dimulai.
A. rumah Latar peristiwa percakapan kedua siswa pada kutipan drama di atas
B. jalan adalah ..
A. Sore hari saat pelajaran
C. bis kota
B. Pagi hari saat pelajaran
D. rumah sakit C. Siang hari di depan kelas
D. Di depan kelas saat pelajaran
4. Suami : Dompet itu telah kau bawa?
Istri : Ya.
Suami : Tak seorang pun mengikutimu?
Istri : Tidak. Aku sendiri pun tak bisa lari cepat.
Suami: […] 6. Raka : Tolooong..., ada yang mau jahat kepada saya!
Istri : Tidak ada yang mendorong. Wanita tua itu menaruh dompet Orang-orang : Mana ... mana orangnya?
di dekat jariku. Ketika aku mengambilnya dia tidak Zidan : Ayo kita hajar biar kapok!
memperhatikanku. Adit : Jangan, ...jangan!Kita tak boleh main hakim sendiri.
Kalimat yang tepat untuk melengkapi dialog drama di atas adalah … Bukti watak Adit dalam kutipan drama tersebut bijaksana, adalah...
A. Apa yang mendorongmu melakukan semua itu? A. Dia mau membantu orang yang berbuat jahat.
B. Apakah kamu tidak takut pada orang itu? B. Dia memberitahukan orang yang berbuat jahat.
C. Siapa yang hendak melakukan semua itu? C. Dia mengajak orang-orang untuk menghajar perampok.
D. Tidak ada yang melihatmu waktu itu kan? D.Dia mengingatkan orang-orang untuk tidak main hakim sendiri.

7. Dr. Hakim : Bicaralah seperti laki-laki, sahabatku Luqman! Siapa


5. Indra : Ndri, sudah beres masalahmu dengan Budi? yang salah katamu? Kamu tahu bahwa semua salah ada
Andri : Ah, bodo amat, Ndra. Dia memang harus dihabisin. padamu ada pada pendirian hidupmu. Bukan pada
Indra : Lho, gimana sih? Bicaralah baik-baik dengannya. Dia kan
nasibmu.
teman kita juga.
Dr. Lukman : Tetapi semuanya aku kerjakan buat kebaikan juga.
Andri : Maunya sih gitu. Tetapi kayanya dia keras kepala.
Indra : Ah, enggak, asal kamu memahami karakter dia. Dr. Hakim : Ya, bagaikan buat dirimu sendiri. Pernahkah kamu
pikirkan istri dan anakmu di rumah? Tidak ada kawan,
tidak ada kasih sayang darimu. Mereka seperti burung
dalam sangkar emas. Sepi kelaparan dan kehausan di 9. Teks samping atau petunjuk teknis dalam drama nomor
tengah-tengah kemakmuran. ditunjukkan oleh kalimat ke …
Dr. Lukman : Tetapi aku sudah beri mereka cukup. Segala yang A. (1)dan (2)
B. (2) dan (3)
diminta kuturuti.
C. (3) dan (4)
Dr. Hakim : Susah bicara sama kamu. [...] D. (4) dan (1)
Dr. Lukman : Tapi uang juga penentu kebahagiaan.
Kalimat yang tepat untuk melengkapi drama di atas adalah… 10. Retha : "Ini pasti ulah Ali. la balas dendam kepada kita." (Sambil
A. Memang istri dan anakmu tidak hanya butuh uang. Dengan membersihkan kelas).
uangmu apapun yang diinginkan akan didapatnya. Rizky : "Rasanya hal itu tak mungkin. Ali bukan tipe anak
B. Memang mencari uang adalah wajib buat kita, mencari uang pendendam. Setahuku selama ini ia baik."
untuk menafkahi keluarga, dan untuk membahagiakan mereka. Retha : "Mungkin juga, buktinya kelas kita sekarang kotor. Mungkin
Itu bagus Lukman, bagus! sepulang sekolah kemarin ia sengaja membuang sampah-
C. Memang mencari uang nomor satu. Uang dapat menyelesaikan sampah ini."
segalanya. Dengan uang kita dapat menggenggam dunia ini. Isi dialog tersebut adalah ....
D.Bagimu uang adalah segalanya. Bahagia tidak bisa dibeli, A. memperdebatkan kelas yang kotor
Lukman! Kasih tidak bisa ditawar. Cinta dan kasih hanya bisa B. prasangka buruk terhadap Ali
dibeli dengan cinta kasih. C. Ali termasuk anak pendendam
D. kekecewaan Retha kepada Ali
8. (1) Ayah : (membuka pintu) ”Dari mana saja kau? Menyusahkan
11. Tapi semua itu tak membuat Nia gembira. Ada beban yang
orang tua!”
(2) Farah : ”Maaf Pak, jangan marah dahulu!” (duduk di kursi) mengganjal di hatinya.
(3) Ibu : ”Ada apa, mengapa pulang sampai malam, biasanya kalau “Kenapa aku mau diangkat oleh orang lain? Padahal aku berharap
ada perlu minta izin? Coba ceritakan!” agar orang tuaku yang datang mengambilku. Kenapa orang tua
(4) Farah: ”Sepulang sekolah, saya menjumpai Dea teman sekelas kandungku tidak datang? Kenapa justru orang lain yang
saya, mendapat kecelakaan. Aduh saya hampir pingsan. Dea mengambilku?” Tanya Nia dalam hatinya.
terkulai tak sadar, tangan dan kepalanya berlumuran darah,” Jika diubah menjadi teks drama, maka cuplikan tersebut dapat
Dialog kutipan drama tersebut yang menunjukkan suasana marah ditulis dalam bentuk …
terdapat pada nomor .... A. prolog
A. (1) B. monolog
B. (2) C. dialog
C. (3) D. epilog
D. (4)
12. Perhatikan kutipan naskah drama berikut! Dialog kutipan drama tersebut yang menunjukkan suasana marah
1) Afnan : (gelisah sekali) "Aduh, bagaimana ini? Tugas harus terdapat pada nomor ....
dikumpulkan sekarang listrik mati juga! A. (1) C. (3)
2) Syifa: (memperhatikan Afnan, menggeleng-geleng) [...] B. (2) D. (4)
3) Marsha : "Iya, ada apa kamu gelisah sekali?
4) Afnan : [….]
5) Marsha: Oh, begitu. Ya, sudah sabar saja!
Dialog yang tepat untuk melengkapi kutipan naskah drama tersebut
adalah .... Bacalah kutipan drama berikut untuk menjawab soal nomor 15-16!
A. (2) Tidak ada apa-apa, Kak. (DI RUANG TENGAH TERJADI PERDEBATAN SERU)
(4) Lagi kesel aja, lampu mati. Faiz : Mengapa begitu banyak kata harus, jangan, wajib? Saya tidak
B. (2) Kamu kenapa, Nan? sanggup memenuhinya. Jangan paksa saya! Saya tidak berbakat
(4) Aku tidak gelisah; aku hanya gerah menjadi orang berdarah dingin. Saya tidak berani menjadi kejam,
C. (2) Kamu kenapa, Afnan? penuh tipu daya. Saya ingin jadi saya sendiri.
(4) Aku mau ngumpulin tugas tapi listrik mati. Mario : Maksudnya?
D. (2) Afnan mau ke mana? Faiz : Ayah adalah ayah. Dengan bakat besar untuk menciptakan
(4) Ini, Kak. Aku mau ngetik tugas. usaha, mencetak uang. Saya adalah saya, bodoh, dan dungu,
bahkan menghitung recehan pun tak mampu.
13. Petunjuk teknis atau petunjuk laku pada kutipan drama nomor 36 Mario : Bukan, Bukan itu. Apa maksudnya berdarah dingin, kejam,
terdapat pada kalimat ke … penuh tipu daya? Ke mana arah pembicaraanmu itu?
A. 1 dan 2 Faiz : Saya tidak ingin membeberkan apa yang saya tahu.
B. 3 dan 4
C. 1 dan 3
D.2 dan 4 15. Latar tempat dalam kutipan drama di atas adalah …
A. ruang tamu C. ruang tengah
14. (1) Ayah : (membuka pintu) ”Dari mana saja kamu? Menyusahkan B. dapur D. depan rumah
orang tua!”
16. Watak tokoh Faiz dalam penggalan drama di atas adalah …
(2) Rima : ”Maaf Pak, jangan marah dahulu!” (duduk di kursi)
A. tegas C. pemarah
(3) Ibu : ”Ada apa, mengapa pulang sampai malam, biasanya kalau
B. pembangkang D. pemalu
ada perlu minta izin? Coba ceritakan!”
(4) Rima : ”Sepulang sekolah, saya menjumpai Nisa teman sekelas 17. Bacalah kutipan naskah drama berikut!
saya, mendapat kecelakaan. Aduh saya hampir pingsan. Nisa Meski saat itu sangat terik, namun teman-teman Ani tetap datang.
terkulai tak sadar, tangan dan kepalanya berlumuran darah,” Yuni dan Hani : (memberi salam bersama)
Ibu : (datang dari dapur) Teman-teman Ani, ya?
Yuni : Iya Bu, kami teman-teman Ani.
Ibu : Mari masuk! Silakan duduk dulu, Ibu panggilkan Ani,
Ani…! Itu teman-teman kamu sudah datang.
Ani : Hai, yuk langsung aja kita mulai.
Yuni dan Hani : Ayo! Bacalah kutipan drama berikut untuk menjawab soal nomor 19 dan
Latar kutipan naskah drama tersebut adalah .... 20!
A. di ruang tamu, sore hari Gisela : Yang pasti kita jangan kerja kelompok di rumah Ikal, rumah
B. di rumah, siang hari dia kan kecil dan kumuh, lagi pula aku nggak boleh sama
C. di ruang belajar, siang hari
orang tuaku jika bermain ke tempat kumuh seperti itu.
D.di rumah, sore hari
Joe : Iya betul sekali itu, pasti rumah Ikal itu bau banget dan
18. Mas Abu : Hasilnya? Pasukan musuh itu mampus semuanya, sempit sekali, pokoknya aku tidak akan mau kerja kelompok
dan senjata-senjatanya kami rampas semuanya. Ah ... kalau aku
jika di rumah Ikal!
terkenang lagi kepada pertempuran-pertempuran seperti itu,
kadang-kadang aku ingin kembali ke zaman perang revolusi itu. Lucy : Teman-teman, kalian jangan ngomong seperti itu, belum
Sungguh mati Saudara-saudara, bukan sombong. tentu apa yang kalian ucapkan itu benar, lagian tujuan kita
Samsu : (Sambil Makan Kroket) Ya, ya, aku bisa mengerti, sebab
itu untuk belajar bukan bermain!
aku pun begitu juga.
Sumantri : (Sambil Mengudek Kopi Susunya) Saudara di mana Ikal : Sudahlah tidak apa-apa, perkataan mereka itu benar kok,
ketika itu? kalau kita belajar di tempat yang tidak nyaman kan juga
Samsu : Saya? Saya ketika itu berada di lereng Gunung memengaruhi tugas kita juga
Galunggung. Saya pun memimpin satu pasukan.
Joe : Bagaimana kalau di rumah Lucy saja!
Inti penggalan naskah drama di atas adalah …
A. Seorang pejuang yang terkenang pada saat perang revolusi. 19. Watak Ikal pada penggalan naskah drama di atas adalah ….
B. Seorang pejuang yang berhasil menghabisi lawanya saat perang
revolusi. A. Iri hati
C. Seorang pemimpin pasukan yang mempimpin perang pada saat B. rendah hati
perang revolusi. C. besar hati
D.Seorang pejuang yang meragukan kepemimpinan pejuang
D.tinggi hati
lainnya pada saat perang revolusi.

20. Suasana yang terjadi dalam penggalan drama di atas adalah ...
A. santai
B. tegang
C. panik
D. tenang

PEMANTAPAN 5

Paragraf dan
Permasalahannya
1. Bacalah teks berikut!

Dampak negatif pemanasan global terus diserukan. Seruan ini


Dampak negatif pemanasan global terus diserukan. Seruan ini
disampaikan setelah Badan Kesehatan Dunia (WHO)
disampaikan setelah Badan Kesehatan Dunia (WHO)
memperingatkan makin hangatnya temperatur udara. Hal ini dapat
memperingatkan makin hangatnya temperatur udara. Hal ini dapat
memicu timbulnya penyakit seperti demam berdarah dengue dan
memicu timbulnya penyakit seperti demam berdarah dengue dan
malaria. Lebih lanjut WHO melaporkan, perubahan iklim
malaria. Lebih lanjut WHO melaporkan, perubahan iklim
menyebabkan lebih dari 150.000 kematian dan lima juta orang
menyebabkan lebih dari 150.000 kematian dan lima juta orang
terserang penyakit setiap tahunnya. Jumlah ini dapat berlipat
terserang penyakit setiap tahunnya. Jumlah ini dapat berlipat
ganda pada tahun 2030.
ganda pada tahun 2030.

Gagasan utama paragraf tersebut adalah …


A. Seruan dampak negatif pemanasan global oleh WHO
B. Peringatan WHO tentang pemanasan global
C. Laporan WHO tentang perubahan iklim
D.Tahun 2030 orang akan mati karena pemanasan global.

2. Bacalah teks berikut!


(1) Banyak hal yang menyebabkan kemacetan lalu lintas. (2)
Pertama, adanya persilangan dengan jalan rel kereta api. (3)
kedua, semakin banyak kendaraan yang berlalu lalang di
jalan. (4) ketiga, banyak jalan digunakan untuk parker
kendaraan dan sebagai tempat para pedagang kaki lima
berjualan.

Kalimat utama paragraf di atas ditunjukkan oleh nomor …


A. (4)
B. (3)
C. (2)
D.(1)

3. Bacalah teks berikut!


Pilihlah salah satu jawaban yang paling tepat! Pada umumnya kucing peliharaan tidak berbahaya bagi manusia
bulunya.
karena tubuhnya yang kecil. Namun, tidak menutup kemungkinan
hewan ini dapat menularkan penyakit. Penyakit yang mungkin
ditularkan di antaranya rabies akibat gigitannya dan gangguan
pernapasan oleh bulu-bulunya.
Informasi tersurat yang terdapat dalam kutipan teks di atas
adalah…
A. Kucing merupakan hewan yang banyak dipelihara manusia
B. Penyakit yang dapat ditularkan kucing di antaranya rabies
C. Tubuh kucing yang kecil membuat hewan ini disukai banyak
orang. Simpulan isi teks tersebut adalah …
D. Kucing dapat menularkan gangguan pernapasan oleh cakarnya. A. Langkah yang dilakukan pengusaha Jatim untuk
mempertahankan usahanya.
4. Bacalah teks berikut! B. Upaya yang dilakukan pelaku usaha di Jawa Timur dalam rangka
mendekati pembeli barang-barang hasil produksi mereka.
Ketua Komisi Nasional perlindungan Anak, Kak Seto,
menyebutkan bahwa yang paling penting adalah upaya menyiasati C. Upaya para pengusaha Porong dalam merelokasi usahanya ke
pendidikan khusus bagi anak pekerja, tetapi bukan bersifat normal beberapa daerah yang jauh dari pusat semburan lumpur Lapindo.
Oleh karena itu, Kemendikbud diharapkan agar membuka pintu D.Langkah para pengusaha di Jawa Timur untuk menghindari
lebih lebar bagi kemungkinan penyamaan kualitas antara anak yang dampak semburan lumpur.
mengikuti pendidikan di sekolah dan di rumah.
6. Bacalah teks berikut!
Tanggapan yang tepat terhadap teks tersebut adalah …
A. Tidak mungkin menuntut anak pekerja untuk bersekolah. (1) Pulau Pari, Kabupaten Kepulauan Seribu merupakan salah satu
tempat dibudidayakanya rumput laut di Indonesia. (2) Budidaya ini
B. Pemerintah agar membuka pendidikan khusus bagi anak pekerja
menjadi sumber penghasilan masyarakat setempat. (3) Rumput laut
C. Khusus bagi pekerja pabrik perlu pendidikan khusus untuk
yang dibudidayakan di sini sangat beragam bentknya. (4) Semua
mereka
dapat hidup karena perawatannya dipandu secara berkala untuk
D.Lebih baik anak-anak
Semburan itu bersekolah
lumpur lapindo daripadakepada
sangat berdampak bekerja.
dunia usaha. melihat perkembangannya.
Para pelaku usaha di Jawa Timur terus berupaya melakukan
terobosan untuk menyiasati dampak berkepanjangan yang
ditimbulkan oleh semburan lumpur lapindo di Sidoarjo. Usaha ini
5. Bacalah teksuntuk
ditempuh berikut!mempertahankan usaha yang menjadi sumber Kalimat utama paragraf tersebut ditandai dengan nomor …
penghidupan mereka. A. (1)
X Umumnya pelaku usaha ini merelokasi usaha mereka ke beberapa B. (2)
daerah seperti Malang, Surabaya, dan Jakarta. Selain agar usaha
mereka tetap eksis, para pengusaha ini sekaligus bertujuan
mendekatkan barang-barang hasil produksi mereka dengan
C. (3)
D. (1)

7. Bacalah teks berikut! Kalimat yang berupa opini ditandai dengan nomor …
A. (1) dan (2) C. (2) dan (3)
Apabila kita masuk ruangan perpustakaan ini, kesan pertama yang B. (1) dan (4) D. (3) dan (4)
terasa adalah bersih, teratur, dan menyenangkan, baik pengaturan
buku-buku maupun dekorasi ruangannya. Semua ini tentu 10. Bacalah teks berikut dengan cermat!
merupakan salah satu unsur, mengapa perpustakaan sekolah ini
Sungai penting untuk kegiatan perikanan. Khususnya, di sungai besar
menjadi juara pertama dan ditetapkan sebagai perpustakaan
yang menjadi tempat hidup beragam ikan dan hewan lainnya. Sungai
teladan se-Jawa Barat tahun ini.
besar ini menjadi tempat nelayan mencari mata pencarian. Manfaat
Pola pengembangan kalimat pada paragraf di atas adalah … sungai dalam pertanian juga tak kalah penting, yakni sebagai sumber
A. Umum-khusus irigasi. Kawasan-kawasan pertanian yang subur mendapatkan
B. Khusus-umum pengairan dari sungai. Belakangan, manusia membendung sungai
C. Umum-khusus-umum sebagai sumber irigasi dan pembangkit tenaga listrik. Pembuatan
D. Khusus-umum-khusus
bendungan ini sangat bermanfaat bagi manusia.

8. Bacalah teks berikut!


Simpulan isi bacaan di atas adalah …
Tempatnya tidak terlalu luas. Hanya sebuah ruang kecil berukuran 3x2
A. Sungai sangat bermanfaat bagi kehidupan manusia.
meter. Jendela kaca yang terletak simetris di ruangan ini begitu asri
B. Sungai sumber mata pencarian nelayan.
menembus pemandangan alam pegunungan. Tempat tidur beralaskan
C. Sungai sebagai sumber kehidupan bagi sebagian manusia.
sprai tokoh kartun Hello Kitty kesukaanku lengkap dengan bonekanya. D. Bendungan bermanfaat bagi kelangsungan hidup.
Di sampingnya, sebuah meja belajar rapi dengan buku-buku pelajaran
dan koleksi teenlit terbitan Gagas Media bacaan favoritku tertata rapi.
Inilah kamar tidurku.

Paragraf di atas termasuk jenis paragraf … 11. Bacalah teks berikut!


A. deskripsi C. argumentasi Penyakit gigi pada anak dapat berdampak pada kesehatan masa
B. eksposisi D. persuasi depannya. Salah satunya adalah struktur gigi yang tak benar
9. Bacalah teks berikut! karena kerusakan gigi terutama pada gigi susu. Sementara itu,
sakit gigi karena gigi berlubang dapat mengganggu aktivitas belajar
Matematika merupakan pelajaran yang diujiannasionalkan di tingkat
sehingga prestasi mereka pun akan menurun.
SMP (1). Banyak siswa yang mengatakan bahwa matematika itu
membosankan (2). Pernyataan ini tidak sepenuhnya benar karena
banyak juga yang berprestasi di bidang matematika (3). Panduan
belajar matematika yang menyenangkan ditulis oleh Brian Bell (1992), Gagasan utama paragraf tersebut adalah …
dalam bukunya banyak sekali variasi permainan berbasis matematis
dan juga teka-teki yang mengasyikan (4).
A. Dampak penyakit gigi pada kesehatan. C. Gubernur se-Indonesia
B. Kerusakan gigi berlubang mengganggu aktivitas belajar D. Gubernur Sulawesi Selatan
C. Sakit gigi berlubang dianggap hal sepele
D.Gigi susu berlubang menjadikan struktur gigi jelek. 4. Penamba
Penambahan lahan sudah disadari menjadi kendala dalam
pengembangan berbagai sektor. Bertambahnya permintaan lahan
Bacalah teks berikut untuk menjawab soal nomor 12 dan 13! dipengaruhi oleh bertambahnya jumlah penduduk. Selain itu,
Gubernur Sulawesi Selatan, Syahrul Yasin Limpo, mengajukan protes meningkatnya kesejahteraan penduduk membutuhkan pengembangan
langsung kepada Presiden Joko Widodo dan para menteri. Ini permukiman untuk tempat tinggal dan pengembangan industri untuk
berlangsung di Istana Bogor Senin 24 November 2014, saat Presiden mencukupi kebutuhan hidup.
Jokowi beserta menteri-menterinya bertemu dengan para gubernur se-
Indonesia. Syahrul mengatakan para menteri Kabinet Kerja seringkali Simpulan isi paragraf tersebut adalah …
tiba-tiba datang ke daerah tanpa menyampaikan pemberitahuan A. Sektor industri tidak berkembang karena ketidaksiapan
terlebih dahulu kepada gubernur. Menurut Syahrul, kedatangan para infrastruktur.
menteri membuat para gubernur kelabakan. Karena mereka tak sempat B. Pembangunan kawasan industri membantu pembangunan
mempersiapkan apapun. Sebelumnya, tak hanya para menteri yang
infrastruktur.
sering turun ke daerah, tetapi juga Presiden Joko Widodo kerap tiba-
C. Dibutuhkan lahan untuk mencapai kesejahteraan pangan.
tiba melakukan kunjungan ke daerah. Pernyataan ini hendaknya
ditanggapi Presiden dan para menterinya dengan baik. D. Penambahan lahan dipengaruhi oleh jumlah penduduk dan
tingkat kesejahteraan.

Bacalah teks berikut untuk soal nomor 15!

2. Tanggapan yang sesuai dengan paragraf di atas adalah … (1) Badan kependudukan PBB atau United Nations Population Fund
A. Sebaiknya presiden dan para menteri berkoordinasi terlebih (UNPF, dulu UNFPA), Rabu 27 Juni 2007, menerbitkan laporan
dahulu dengan kepala daerah sebelum melakukan kunjungan. tentang kependudukan dunia. (2) Laporan tersebut menyebutkan
B. Presiden Joko Widodo dan kabinetnya memang hobi blusukan. lebih dari setengah penduduk dunia atau 3,3 miliar akan tinggal di
C. Kepala daerah tidak perlu banyak berkomentar, presiden berhak kawasan perkotaan pada tahun 2008. (3) Jumlah ini diperkirakan
melakukan apa saja. akan meningkat menjadi lima miliar pada tahun 2030. (4) Penghuni
D.Para Gubernur seharusnya kompak melakukan protes kepada baru perkotaan ini adalah keluarga berpenghasilan rendah. (5) Kalau
presiden dan para menterinya. saja tidak dilakukan langkah responsif, banyak kota yang tidak akan
mampu menampung pertumbuhan cepat tersebut. (6) Penyebabnya
3. Dalam paragraf tersebut penulis berpihak kepada … bukan migrasi, tetapi pertumbuhan alami dengan jumlah kelahiran
A. Presiden lebih banyak daripada jumlah kematian.
B. Menteri
D.Kebersihan lingkungan sekolah akan menjadi penjamin
5. Kalimat yang merupakan opini penulisan terdapat pada nomor ... kesehatan dan keberhasilan warga sekolah.
A. (1) dan (2) C. (3) dan (5)
B. (2) dan (3) D. (4) dan (5) 8. Keluarg
Keluarga menjadi wahana utama pewarisan peradaban dan budaya.
Dalam keluarga dipraktikkan nilai dasar hubungan antarmanusia.
6. Tim nasional Indonesia U-19 dijadwalkan menggelar pemusatan Selain itu dalam keluarga dipraktikkan kesalingpahaman hak dan
latihan di Eropa. Ternyata, ada beberapa menu wajib yang dibawa kewajiban sesama sebagai anggota kelompok. Dengan demikian,
Evan DImas dan kawan-kawan ke salah satu negara benua biru keluarga bukan hanya objek, tetapi mitra atau subjek program
tersebut. Kecap jadi ‘barang’ yang wajib dibawa. Kecap sebagai pembangunan bangsa.
penyedap makanan aman untuk dikonsumsi Timnas U-19.
Pernyataan yang sesuai dengan teks tersebut adalah … Gagasan utama paragraf tersebut adalah …
A. Jadwal Pemusatan latihan Timnas U-19 di Indonesia. A. Keluarga merupakan tempat utama mewariskan adab dan
B. Persiapan yang dibawa Evan Dimas dan kawan-kawan. budaya.
C. Kecap adalah makanan kesukaan seluruh Timnas U-19 B. Keluarga menanamkan nilai dasar hubungan antarmanusia.
D. Tim Nasional Indonesia U-19 dijadwalkan latihan di Eropa C. Keluarga kurang berperan bagi pembangunan bangsa.
7. Kebersihan lingkungan sekolah adalah hal yang tidak bisa D.Manusia sebagai anggota keluarga wajib membangun bangsa.
dipisahkan dari kehidupan sekolah. Dengan kebersihan lingkungan,
warga sekolah menjadi nyaman. Kenyamanan mereka menjadi
factor penunjang keberhasilan belajar bagi siswa. Dengan
kenyamanan itu pula, para guru lancer menjalankan tugasnya. 9. DiDi kawasan Gunung Kidul ekosistem masih terjaga. Di daerah itu
Kebersihan lingkungan sekolah juga mendukung kesehatan warga sungai di bawah tanah airnya melimpah. Di gua dan sekitar sungai
sekolah. Dengan lingkungan yang bersih, sekolah terbebas dari Gagasan utama
masih dihuni paragraf tersebut
sekelompok kelelawar adalah …
dan fitoplankton. Fitoplankton itu
berbagai penyakit. Penyakit berat maupun ringan tidak akan menjadi makanan ikan sehingga
A. Salah satu gua di Gunung Kidul ikan berkembang biak dengan baik.
menyerang warga sekolah. B.Hewan-hewan
Penghuni gua melata atau reptil,
di Gunung Kidulseperti ular, kadal, dan tokek masih
Ringkasan teks tersebut adalah …. C.berkeliaraan. Burung-burung
Tempat ekosistem kecilterjaga
yang masih berkicau, musang berlari-larian,
ayam berkokok, dan berbagai serangga hidup saling pengaruh.
A. Kebersihan lingkungan sekolah membuat warga sekolah menjadi D. Sungai bawah tanah.
nyaman dan sehat.
B. Kebersihan lingkungan tidak terpisahkan dengan kesehatan
Jus alpukat sangat bagus untuk dikonsumsi setiap hari.
warga sekolah yang nyaman. 10. Jus
Terlebih bagi Anda yang bergolongan darah A, zat kandungannya
C. Kebersihan lingkungan sekolah mendukung keberhasilan dan dapat mendorong suasana basa di dalam tubuh. Jus ini
kesehatan seluruh warga sekolah. mengandung lemak tak jenuh yang mudah dicerna oleh tubuh.
Ada dampak positif lemak alpukat bagi tubuh. Pertama, dapat
menurunkan kadar kolesterol darah (LDL). Kedua, mencegah
terjadinya penyakit jantung, kanker, stroke dan darah tinggi.
11. Kalimat utama paragraf tersebut terdapat pada nomor ...
Rangkuman isi bacaan tersebut yang tepat adalah …. A. (1) B. (2) C. (3) D. (4)
A. Jus mengandung lemak tak jenuh dan mudah diserap tubuh.
B. Dampak positif jus dapat menurunkan LDL, stroke, dan darah 12. Makna kata domestik dalam paragraf tersebut adalah ...
tinggi. A. asli Indonesia C. dalam negeri
C. Jus alpukat sangat bagus dikonsumsi karena berdampak positif B. pribumi D. luar negeri
bagi tubuh.
D. Jus alpukat mengandung vitamin dan mudah diserap tubuh. 13. Kalimat penjelas yang tidak mendukung kalimat utama dalam
paragraf tersebut adalah nomor ....
A. (2) B. (3) C. (4) D. (5)

14. Cokelat
Cokelat yang sering meniadi
primadona untuk memulihkan rasa
stres ternyata dapat mencegah
pertumbuhan jerawat yang sedang
Bacalah teks berikut ini untuk menjawab soal nomor 21, 22, dan diderita oleh seseorang. Sebuah
23! penelitian yang dilakukan oleh
mahasiswa kedokteran di
(1) Objek wisata Pangandaran University of Miami School of
menyediakan transportasi rekreasi untuk Medicine berhasil menemukan
memudahkan wisatawan menikmati cokelat memiliki peran mencegah
keindahan pantai. (2) Wisatawan domestik pertumbuhan jerawat. Responden
maupun mancanegara dapat menggunakan dalam penelitian tersebut terdiri atas
transportasi untuk menikmati keindahan sepuluh pria berusia 18 hingga 35
alam. (3) Di sepanjang tepi Pantai tahun. Mereka diminta memakan
Pangandaran terlihat berjejer perahu untuk cokelat murni sebanyak tiga hingga
disewakan. (4) Dengan biaya Rp 1.000,00 empat ons. Setelah itu mereka harus
saja per orang, para wisatawan dapat diet selama seminggu. Selama proses
berputar di sekitar pantai dan menikmati diet inilah ditemukan hormon jerawat
keindahan taman laut. (5) Para pedagang yang kian hari kian memburuk.
ikut meramaikan situasi pantai
Pangandaran.
Simpulan isi paragraf tersebut adalah ....
A. Cokelat baik untuk memulihkan stres.
B. Cokelat diteliti oleh mahasiswa kedokteran di University of
Miami berperan mencegah pertumbuhan jerawat.
C. Responden mengalami pertumbuhan jerawat yang kian
memburuk.
D.Coklat dapat membantu proses diet selama seminggu

15. Hawa yang lembap memungkinkan banyak jenis jamur tumbuh


subur di lereng-lereng bukit dan gunung. Akibatnya, tidak sedikit
warga beramai-ramai memetik jamur liar itu untuk dimasak. Namun,
pakar memperingatkan jangan sembarangan memetik jamur liar
untuk dimasak karena sebagian jamur liar dapat membahayakan
jiwa.
Informasi yang tidak sesuai dengan isi teks di atas adalah …
A. Warga mengira tidak ada persoalan dengan jamur dan hawa
lembap.
B. Masyarakat mengonsumsi jamur liar untuk dimasak.
C. Hawa yang lembut memungkinkan jamur liar tumbuh dimana-
mana.
D.Pakar memperingatkan warga untuk tidak mengonsumsi jamur
liar.

***
PEMANTAPAN 6

Teks Puisi
Sumber pengabdianku kepadamu
Kalau hari minggu engkau datang ke rumahku
Aku takut anak-anakku
Kursi-kursi tua yang di sana
Dan meja tulis sederhana
Amanat dari penggalan puisi di atas adalah …
A. Tuhan selalu menerima manusia yang bertaubat
Pilihlah salah satu jawaban yang paling tepat! B. Guru adalah pengganti orang tua
C. Jangan menutup diri
1. HAMPA D.Muliakanlah guru sebagai pahlawan tanpa tanda jasa
Sepi di luar
Sepi menekan mendesak 3. Pedang di kanan dan keris di kiri
Lurus kaku pohonan Berselempang semangat yang tak bisa mati
Tak bergerak Ini barisan tak bergandengan berpalu
Sampai ke puncak Kepercayaan tanda menyerbu
Sepi memagut Penggalan puisi di atas melukiskan …
Segala menanti, menanti, menanti A. Semangat yang terbelenggu
Sepi B. Sikap pasrah terhadap Tuhan
Tambah ini menanti jadi mencekik C. Perjuangan tanpa senjata
Memberat mencekung punda D. Semangat perjuangan yang pantang menyerah
Sampai binasa segala
Belum apa-apa 4. Engkau selalu bersiaga
Karya: Chairil Anwar Menjaga sarang diterjang lawan
Amanat yang tepat dalam puisi tersebut adalah... Kau terkam si Kulit bundar
Kau halau ke tengah lapang
A. Hendaknya jangan membuat seseorang harus menunggu
Puisi tersebut berisi tentang …
B. Menunggu adalah pekerjaan yang sangat membosankan A. Pemain bola
C. Menunggu adalah pekerjaan yang menyedihkan manusia B. Penjaga gawang
D.Hendaknya seseorang menghindari kebiasaan menunggu. C. Si Kulit Bundar
D.Macan Lapangan
2. Dari seorang guru kepada murid-muridnya
Adakah yang kupunya anak-anakku Bacalah puisi berikut untuk menjawab soal nomor 5 s.d. 6!
Selain buku-buku dan sedikit ilmu Gadis Peminta-minta
…..
Ingin aku ikut, gadis kecil berkaleng kecil 8. Anak Yang Angkuh
Pulang ke bawah jembatan yang melulur sosok Betapa dinginnya air sungai
Hidup dari kehidupan angan-angan yang bergemerlapan Dinginnya! Dinginnya!
Gembira dari kemayaan riang Betapa dinginnya daging duka
Yang membaluti tulang-tulangku.
Duniamu yang lebih tinggi dari menara katedral
..... (Karya W.S. Rendra)
Melintas-lintas di atas air kotor Puisi tersebut menimbulkan citraan ...
Tapi yang begitu kau hafal a. penglihatan C. Pendengaran
Jiwa begitu murni, terlalu murni b. Penciuman D. perabaan
Untuk bisa membagi dukaku
9. KAU
5. Amanat kutipan puisi tersebut adalah … Kubelai dengan mesra
A. Hidup pengemis penuh liku-liku Harummu mengundang
B. Hiasilah hidup ini dengan menara katedral Gairah jiwa
C. Jangan iri terhadap kehidupan orang lain Oh … bungaku
D. Sudah selayaknya kita bermimpi Kau cantik jelita
Larik kedua puisi di samping menggunakan citraan …
6. Suasana yang tergambar pada puisi tersebut adalah … A. pendengaran C. perabaan
A. gembira B. penciuman D. Perasa
B. haru
C. mencekam 10. O, bebaskan daku dari kesulitan
D. menakutkan Biarkan rembulan menyapaku
Merambah gunung menyeberangi lautan
Dan angin yang menghiburku
Larik yang bermajas sama pada puisi tersebut adalah …
7. Ke Blora ia akan menikmati A. 1 dan 2 C. 2 dan 3
Ke Blora ia akan kembali bermimpi B. 1 dan 3 D. 2 dan 4
Masa kanak dan cinta yang gagal
Ia terlalu hafal 11. Kala malam berganti waktu
Citraan yang terdapat pada puisi di atas adalah … Dewi malam mulai sirna
A. penglihatan C. pendengaran Kutinggalkan mimpi buruk
B. perasaan D. Penciuman Tuk menyambut mentari pagi
Majas yang terdapat pada baris kedua puisi di atas adalah … D. sungai bernyanyi
A. personifikasi C. pleonasme
B. metafora D. hiperbola 14. Kapal laut
Lihatlah sebuah titik
Di tengah laut nan jauh
Titik itu terus menggelinding
Kuat tiada …
Walau di terjang omabk diterpa angin
12. Suara bedug menusuk kalbu Kata yang tepat untuk melengkapi kutipan puisi tersebut agar
Suara adzan memanggil hati berirama sama dengan baris sebelumnya adalah …
…………. a. gentar C. menjauh
Di dalam dada rasa berrgejolak b. berkelit D. bergeming
Larik bermajas yang sesuai untuk melengkapi baris ketiga puisi
tersebut adalah … 15. Mengapa bunga harus mekar (1)
A. Hati menangis teringat dosa Setelah kumbang dapatkan madu(2)
B. sedih terasa di dada Mengapa kumbang harus ingkar (3)
C. Rasa pilu tidak menentu … (4)
D. Harapan terasa hampa Agar baris ketiga dan keempat puisi tersebut berirama sama,baris
keempat yang tepat adalah …
13. Hati pun rapuh A. Setelah bunga jadi layu
Bagai benang laba-laba B. Setelah bunga tak lagi mekar
Cahaya memucat C. Setelah bunga tersipu malu
Bulan April gemetar D. Setelah menggoda sang kumbang
Dalam angin dan sayup-sayupnya
Lembut bagai bulu 16. Aku
…. Kalau sampai waktuku
Dalam kedamaian yang hening Ku mau tak seorang kan merayu
Kalimat bermajas yang tepat untuk melengkapi puisi rumpang di Tidak juga kau
atas adalah … Tak perlu sedu sedan itu
A. aku bernyanyi …
Suasana dalam puisi tersebut adalah …
B. sungai berair
A. ketegaran seseorang
C. dia berdendang
B. keluh kesah perjuangan
C. suatu penyesalan
D. perasaan demdam

17. Oktober
Karya: Mardi Luhung

Ketika kau tertidur napas halusmu melayang menebus jalusi.


Dan napasku yang tergopoh pun menyusulnya Agar dapat
menjaganya sampai nanti kau terbangun ….
Puisi di atas menggunakan majas ....
A. metafora Cermati puisi berikut dengan saksama kemudian kerjakan soal
B. personifikasi nomor 19 dan 20!
C. alegori Jangan kau pergi!
D. antitesis Kurindu canda dan tawamu
Denganmu kubisa berharap...
18. Jemu aku dengan bicaramu. Sahabat...
Kemakmuran, keadilan, kebahagiaan Ke mana kuharus mencari?
Kau pergi tanpa pesan
Sudah sepuluh tahun engkau bicara
Maafkan segala salahku
Aku masih tak punya celana
Budak kurus pengangkut sampah 19. Tema puisi tersebut adalah ...
Tema penggalan puisi di atas adalah …. A. Kesepian
A. keadilan B. Kegelisahan
B. kejemuan C. kerinduan
C. kritik sosial D. kedamaian
D. kemelaratan
20. Suasana yang tergambar pada puisi tersebut adalah ...
A. sedih
B. gembira
C. haru
D. gugup

***
PEMANTAPAN 7

Teks Prosedur dan


Teks Observasi
(3) Panaskan minyak goreng, goreng kroket sampai matang, angkat
tiriskan.
(4) Panaskan margarin, tumis bawang bombay sampai layu.
Pilihlah salah satu jawaban yang paling tepat! Kemudian masukkan wortel dan setengah bagian susu cair,
masak sampai mengental.
1. (1) Pisahkan cucian yang mudah luntur warnanya! (5) Tambahkan garam, merica, bumbu kaldu, sisa susu, dan
(2) Larutkan 30 gram deterjen ke dalam 10 liter air! maizena cair. Masak sampai semua bahan matang, angkat dan
(3) …. dinginkan.
(4) Kucek kemudian bilas dengan air hingga bersih! Urutan pembuatan kroket yang benar adalah …
(5) Jemur pakaian hingga kering! A. 4 – 5 – 1 – 2 – 3
Untuk melengkapi petunjuk mencuci pakaian di atas adalah … B. 4 – 1 – 5 – 2 – 3
A. Campurkan deterjen dengan air! C. 2 – 4 – 5 – 1 – 3
B. Rendam cucian selama 30 menit! D. 2 – 5 – 1 – 4 – 3
C. Campurkan semua cucian!
D. Bilas hingga bersih! 4. Petunjuk cara menyimak
1) Tangkap semua suara yang terdengar
2. Petunjuk Pembuatan Roti Manis: 2) Pusatkan perhatian pada sumber suara
1) Larutkan gula dan garam dalam air! 3) Catatlah hal-hal penting
2) Masukkan susu bubuk dalam tepung yang sudah disediakan! 4) Ciptakan suasana hening
3) Campurkan semuanya beserta ragi pengembang yang sudah Petunjuk cara menyimak yang benar adalah …
dilarutkan dalam air hangat. A. 4 – 2 – 3 – 1
4) … B. 2 – 4 – 3 – 1
5) Masukkan margarine dan kocok, kemudian simpan adonan C. 2 – 4 – 1 – 3
hingga mengembang. D.4 – 2 – 1 – 3
Kalimat yang tepat untuk melengkapi petunjuk di atas adalah …
A. Aduk adonan sampai merata! 5. Cara membuat saus tomat:
B. Cairkan sedikit margarin dan masukkan ke dalam adonan! 1) Tomat dicuci sampai bersih, kemudian kukus selama lima menit!
C. Satukan adonan pertama sampai ketiga! 2) Saus tomat dimasukkan ke dalam botol!
D. Tambahkan air sedikit demi sedikit!
3) Selama proses penghancuran tambahkan bumbu-bumbu sesuai
3. Cara membuat kroket dengan selera!
(1) Masukkan daging ayam, makaroni, jamur merang dan baso 4) Pilihlah tomat yang tua dan merah!
ikan. 5) Setelah menjadi bubur yang halus kemudian masak!
(2) Ambil dua sendok adonan, bentuk bulat memanjang, masukan ke 6) Tomat yang sudah dikukus kemudian dikupas lalu
dalam kocokan telur, kemudian gulingkan ke dalam tepung panir. blender/hancurkan!
Urutan yang tepat untuk membuat saus tomat yang tepat adalah …
A. 1-4-3-6-2-5
B. 4-1-6-3-5-2
C. 3-6-5-1-4-2
D. 6-2-4-6-5-1

6. Yang termasuk struktur teks prosedur adalah ….


C. tujuan – material – langkah-langkah
D. langkah-langkah – prosedur – tujuan
E. prosedur – material – alat dan bahan Urutan petunjuk menyalakan televisi yang tepat adalah …
F. tujuan – alat dan bahan – material A. 4 – 2 – 3 – 1 – 5 C. 5 – 3 – 1 – 2 – 4
B. 1 – 4 – 2 – 3 – 5 D. 2 – 3 – 1 –5 – 4

7. Cermatilah petunjuk pembuatan cincau lidah buaya berikut, 9. Cara membuat Jus Mangga
1) Siapkan irisan daging daun lidah buaya yang telah dicuci bersih Kupas buah mangga yang sudah
secukupnya. masak dan lepaskan daging
2) Masukkan irisan daging daun lidah buaya dalam santan buahnya dari bijinya!
3) Siapkan air santan kelapa diberi gula aren secukupnya dalam […]
panci Tambahkan es batu!
4) Tuanglah hasil rebusan dalam mangkuk atau gelas. Tambahkan gula pasir dan susu
5) Rebuslah daging lidah buaya dan santan tersebut sampai secukupnya!
mendidih. […]
6) Cincau daging daun lidah buaya siap disajikan Kalimat yang tepat untuk bagian rumpang petunjuk tersebut adalah
Hidupkan blender selama tiga menit!
Urutan petunjuk pembuatan cincau lidah buaya agar menjadi …
Tuangkan jus mangga ke dalam lima
teks prosedur yang tepat adalah …. gelas!
A. [2] Masukan ke dalam blender!
A. (1) - (2) - (3) - (4) - (5) - (6) [5] beri air matang dingin secukupnya!
B. Tekan
(1) (1) - (3)tombol
- (2) - (4) - (5) remote
nomor - (6) control! B. [2] beri air matang dingin secukupnya!
C. Tekan
(2) (1) - (3)tombol
- (2) - (5) - (4)
main - (6)
power! [5] Masukan ke dalam blender!
D. Indikator
(3) (1) - (3) - (2) - (6) atau
power - (4) -stand
(5) by akan C. [2] tambahkan butiran es batu lagi!
[5] siapkan lima gelas yang sudah dicuci!
berwarna merah! D.[2] hidupkan blender!
8. (4)
Petunjuk menyalakan
Hubungkan kabel televisi
daya listrik [5] beri air matang dingin sebanyak-banyaknya!
pesawat tv ke stopkontak!
(5) Selamat menonton! 10. Bacalah petunjuk kerja berikut dengan cermat!
Cara mengaktifkan kartu ponsel Anda : meninggal di tempat. Setelah melepaskan tembakan, pelaku
1) Ponsel mati/OFF. langsung melarikan diri.
2) Masukkan kartu ponsel Anda. Teks di atas termasuk jenis laporan ....
3) Hidupkan ponsel Anda/ON. A. perjalanan
4) Gosoklah panel pada kartu USER ID dengan hati-hati untuk B. peristiwa
mengetahui PIN Anda. C. kegiatan
5) Masukkan kode PIN yang berjumlah 4 (empat) angka secara D. penelitian
benar dengan cara: ketik (kode PIN) kemudian tekan #OK/Yes.
6) Tunggu kode jaringan Satelindo GSM muncul di layar ponsel. 12. Bacalah kedua teks berikut!
7) Lakukan panggilan telepon ke nomor apa saja (bebas pulsa atau Teks 1
bukan bebas pulsa) dan dengarkan petunjuk yang disampaikan Minggu dini hari, gempa berkekuatan
untuk memilih bahasa pengantar layanan Anda, lalu pastikan 6,0 SR melanda California. Gempa ini
panggilan Anda dijawab. menyebabkan tiga orang terluka.
8) Kartu ponsel Anda telah aktif dan siap digunakan Paginya korban dapat dievakuasi. Siang
Pernyataan yang tidak sesuai dengan cara mengaktifkan kartu hari bantuan tiba di lokasi. Hingga
ponsel di atas adalah ... malam penanganan korban terus
A. Kartu ponsel dimasukkan dalam keadaan ponsel mati dilakukan.
B. Untuk mengetahui PIN dapat dilakukan dengan menggosok panel Teks 2
pada kartu USER ID. Kota Johannesburg, Afrika Selatan,
C. Kode PIN dimasukkan dengan cara mengetik kode PIN kemudian diguncang gempa dengan kekuatan 6,3
tekan #ON. SR. pusat gempa berada di Kota
D.Setelah jaringan Satelindo GSM muncul, dapat dilakukan Orkney yang berjarak 170 kilometer
panggilan ke nomor apa saja. dari Johannesburg. Sebuah gedung
kosong runuh dan menimpa korban.
Perbedaan pola penyajian kedua teks tersebut adalah …
Teks 1 Teks 2
A. Urutan Urutan waktu
peristiwa
11. Hari Minggu siang, 2 Februari 2011 saat korban sedang membuka
B. Urutan umum- Urutan khusus-
praktik pengobatan pada hari Peukan Desa Pasie Mali, Kecamatan
khusus umum
Woyla, Aceh Barat tiba-tiba datang dua orang tak dikenal C. Urutan waktu Urutan tempat
melepaskan tembakan ke arah korban yang mengenai bagian dada, D. Urutan sebab- Urutan akibat -
perut, serta lengan kiri hingga menyebabkan korban jatuh dan akibat sebab
smgpt strategis, tempat ini banyak dikunjungi oleh wisatawm
domestik Di taman ini terrdapat gedung oval, gedung budaya, dan
tempat bcmain mak-anak. Dalam gedung oval pengunjung dapat
belajar tentang sains dm pengetahuan umum. Di gedung budaya
pengunjung dapat menikmati aneka budaya yang ada di
Yogyakarta, sedangkan di tempat bermain dusajikan bermacam-
macam permainan anak.
13. Cermatilah kalimat-kalimat berikut! Isi laporan di atas adalah....
A. Objek wisata Taman Pintar Yogyakarta.
(1) Situasi semakin mencekam, sebab B. Kunjungan wisatawan ke Taman Pintar.
puncak Merapi sama sekali tidak C. Objek wisata di Yogjakarta
terlihat karena tertutup awan tebal. D. Pengunjung ke Objek Wisata di Yogja.
(2) Seismograf (alat pendeteksi gempa)
terus dipasang untuk mengetahui
aktivitas Merapi sudah berkurang
atau semakin tinggi.
(3) Kami menyaksikan situasi letusan
pertama pukul 06.45 mengarah ke
barat daya dari lokasi pengamatan,
(4) Sedangkan wedhus gembel itu terlihat 16. Siswa kelas VI SD. Ngringo 3 melakukan penelitian tentang
sangat jelas menyusuri lereng menuju kebersihan sungai hilir. Penelitian dilakukan mulai tanggal 8–14 Mei
hulu Sungai Krasak, Gendol, dan 2011. Hasil penelitian menunjukkan bahwa kondisi Sungai Hilir
Boyong. sangat memprihatinkan. Sungai yang bersih dan sehat sudah jarang
(5) Akan tetapi, masyarakat Kecamatan ditemui. Sungai yang sehat dan bersih hanya terdapat di desa-desa
Selo masih tenang-tenang saja. yang jauh dari industri dan pabrik.
Isi laporan penelitian di atas adalah .…
Urutan laporan yang tepat adalah … A. Penelitian siswa SD Ngringo 3
A. (3)-(2)-(1)-(5)-(4) B. Limbah pabrik mencemari sungai
B. (3)-(2)-(1)-(4)-(5) C. Kebersihan Sungai Hilir
C. (2)-(3)-(1)-(5)-(4) D. Sungai yang bersih jarang ditemui
D. (2)-(4)-(1)-(5)-(3.)
17. Hasil pengamatan siswa SD Mandala selama satu minggu, mulai
14. Salah satu objek wisata di Yoryakarta adalah Tamam Pintar. tanggal 9-15 Februari 2009 melaporkan kondisi Bengawan Solo.
Tempat wisata ini terletak di Jalan Malioboro. Kareira trmpeya Debit airnya cukup. Airnya kotor bercampur dengan limbah pabrik.
Isi laporan pengamatan tersebut adalah…
A. Pencemaran di sungai Bengawan Solo.
B. Limbah industri di sekitar Bengawan Solo.
C. Pabrik di sekitar Bengawan Solo membuang limbah.
D.Kondisi Sungai Bengawan Solo saat ini.

18. Cermatilah kalimat-kalimat berikut….


1) Kelompok Kami memutuskan untuk mengamati green house
sekolah,
2) Tanaman-tanaman tersebut, antara lain sirih merah, serai,
kemangi, jambu biji, dan susak. 19. Kesalahan penggunaan kata dalam kalimat pertama teks laporan
3) Pagi tadi, kami ditugaskan oleh guru bahasa Indonesia untuk tersebut adalah ....
melakukan pengamatan di lingkungan sekolah. A. kata "berbeda-beda" seharusnya ditulis "berbeda"
4) Sungguh pembelajaran yang menyenangkan hari itu. B. kata “para” yang mubazir
5) Hasil idenrifikasi tersebut kami presentasikan di depan C. kata “puluhan ribu” seharusnya dihilangkan
kelompok lain D. kata “tetapi” seharusnya diganti “namun”
6) Di green house, kami mengidentifikasi sejumlah tanaman yang
bisa dijadikan obat 20. Kesalahan ejaan pada kalimat terakhir teks laporan tersebut
Urutan kalimat tersebut agar menjadi teks laporan yang tepat adalah ....
adalah .... A. kata "kembali” seharusnya diganti “pulang”
B. kata "walaupun" seharusnya diganti "meskipun"
A. (1) - (2) - (5) - (6) - (3) - (4) C. kata "kembali" seharusnya diganti ''dikembalikan"
B. (1) - (3) ribu
Puluhan - (6) - ( 4)
para- (2) transmigran
- (5) D.kata "asalnya" seharusnya diganti "asal mereka"
C. (3) -tinggal
yang (1) - (2)di
- (4) - (6) - (5)wilayah di
berbagai
D. (3) - (1) - (6)Selatan
Kalimantan - (2) - (5) - berbeda-beda
(4)
nasibnya. Ada yang sukses, tetapi
Cermatilah
tidak sedikit kutipan
yang teksgagal. laporan berikut untuk mengerjakan
Di antara
soal nomoryang
mereka 19-20!
gagal, ada yang tetap
bersabar sambil berusaha lebih keras
memperbaiki kehidupan mereka. Bagi
yang tidak mau bersabar, banyak
yang kembali ke Jawa walaupun
mereka tidak memiliki apa-apa lagi di
daerah asalnya.
21. (1) Dari sisi cerita, Guntur Soeharjanto lewat “Assalamu’alaikum lima puluh meter di bawah permukaan air laut dengan suhu tertentu,
Beijing” lagi-lagi cukup berani untuk menampilkan sebuah kisah serta di air jernih yang tidak terkena polusi. Di samping terumbu
cinta yang di dalamnya terdapat berbagai perbedaan dan karang, Taman Nasional Bunaken juga dihuni beragam jenis ikan,
pertentangan budaya dan agama. (2) Di film ini, Guntur juga seperti ikan kuda gusumi, oci putih, lolosi ekor kuning, goropa. Ikan
kembali menggambarkan kehidupan muslim sebagai minoritas di laut Indonesia yang sudah dijadikan industri antara lain ikan tuna,
Tiongkok serta cara mereka bias membaur di tengah-tengah tongkol, tenggiri, kerapu, baronang.
masyarakat yang sangat heterogen. (3) Hampir semua produser film
di Indonesia mulai berlomba memproduksi film. (4) Meski memiliki 23. Ide pokok paragraf di atas adalah ....
cerita yang cukup serius dan melankolis, kehadiran Deddy A. Terumbu karang hidup di air jernih yang tidak terkena polusi.
Mahendra Desta dengan humornya membuat emosi yang ada di B. Terumbu karang di Taman Nasional Bunaken sangat banyak
film “Assalamu’alaikum Beijing” ini menjadi dinamis. jenisnya.
C. Taman Nasional Bunaken tidak hanya di huni terumbu karang
Kalimat yang tidak padu dalam teks ulasan di atas ditujukkan
tetapi juga jenis ikan.
dengan nomor …
D.Ikan laut Indonesia yang sudah dijadikan industri antara lain ikan
A. (1) C. (3)
B. (2) D. (4) tuna, tongkol, tenggiri, kerapu, baronang.

24. Berdasarkan struktur teks hasil observasi, paragraf di atas


22. (1) Dampak yang dihasilkan oleh sampah sangatlah banyak. (2)
termasuk bagian ....
sampah dapat menyebabkan lingkungan menjadi kotor. (3) Selain
A. definisi umum
itu, sampah dapat menjadi sumber penyakit. (4) Kita menginginkan
B. deskripsi umum
kesejahteraan. (5) oleh karena itu, sampah seharusnya diolah C. definisi bagian
menjadi kompos. D.definisi manfaat
Perbaikan paragaraf yang tidak padu di atas adalah ,…
A. Menghilangkan kalimat ke (4) 25. Langkah-langkah menyusun teks hasil observasi
B. Menghilangkan kalimat ke (5) 1) Menjabarkan deskripsi unsur-unsur penting dari objek yang
C. Mengganti diolah dengan dikemas diamati
D.Mengganti selain itu dengan kecuali itu 2) Mengamati objek
3) Memilih objek pengamatan
Bacalah teks observasi berikut untuk menjawab soal nomor 23 dan 4) Menentukan judul
24! Urutan yang benar menyusun teks hasil observasi adalah ....
Terumbu karang di Taman Nasional Bunaken sangat banyak A. 1, 2, 3, 4 C. 3, 4, 2, 1
jenisnya. Terumbu karang ini hidup di pantai atau daerah yang B. 3, 2, 4, 1 D. 4, 3, 2, 1
terkena sinar matahari dan hidup di perairan yang berada kurang lebih
***

PEMANTAPAN 8

Teks Biografi dan


Teks Deskripsi
Pilihlah salah satu jawaban yang paling tepat!

1.Amir Hamzah
Bacalah memulai
kutipan kepenyairannya
biografi berikut ini!
pada usia yang masih belia. Ketika ia
masih duduk di bangku AMS, puisi-
puisinya teah dimuat majalah Timbul
dan Panji Pustaka. Amir Hamzah terus
menulis hingga ia menjadi mahasiswa
di Sekolah Hakim Tinggi Jakarta. Amir
Hamzah mendapat julukan raja Penyair
Pujangga Baru oleh H.B. Yasin. Masih
menurut H.B Yasin, Amir Hamzah
adalah penyair yang sangat produktif.
Keistimewaan tokoh pada kutipan biografi tersebut adalah …
A. Memulai kepenyairannya pada usia muda
B. Puisinya telah dimuat di majalah Timbul
C. Duduk di bangku AMS
D. Mendapat julukan Raja Penyair Pujangga Baru

2. Bacalah kutipan biografi berikut! 3. Keistimewaan tokoh sesuai dengan isi biografi tersebut adalah …
A. Suka membaca dan belajar sendiri
Bung Hatta sangat aktif
Bung Hatta sangat aktif B. Memiliki kualitas tulisannya
melaksanakan tugasnya tugasnya sebagai
sebagai C. Menentang segala tekanan jepang
melaksanakan
bendahara didi suatu
suatu perkumpulan
perkumpulan D.Pelopor penyair angkatan 45.
bendahara
pemuda Sumatera
Sumatera didi Padang.
Padang. Yang
Yang
pemuda
bernama, JongJong Sumatranen
Sumatranen Bond.
Bond. 4. Sifat yang baik untuk diteladani sesuai dengan biografi tokoh
bernama,
Tetapi, selain
selain aktif
aktif dalam
dalampergerakan
pergerakan tersebut adalah …
Tetapi,
daerah, A. Sabar menunggu esai-esainya diterbitkan
iaia juga memikirkan
daerah, juga memikirkan B. Pandai bersandiwara dengan Jepang
penderitaanrakyat
rakyatakibat
akibatpenjajahan.
penjajahan.
penderitaan C. Berani menentang pemerintahan Jepang
D.Rajin membaca dan rajin belajar
Hal yang dapat diteladani dari kutipan biografi Bung Hatta adalah ..
A. Aktif di organisasi daerah dan melaksanakan tugas Negara 5. Struktur bagian teks biografi adalah …
Chairil
denganAnwar
baik. dilahirkan di Medan, A. Orientasi, peristiwa, reorientasi
B. Organisasi
tanggal yang1922.
26 Juli diikutiChairil
adalah Anwar
Jong Sumatrenan Bond
B. Deskripsi umum, identifikasi
C. Selalutipe
adalah memikirkan
pemuda penderitaan rakyat
yang gemar C. Orientasi, tafsiran, evaluasi, rangkuman
membaca dan belajar sendiri, sehinggarakyatnya
D. Aktif berorganisasi dan memikirkan
D.Evaluasi, deskripsi, penegasan ulang
tulisannya memiliki kualitas yang
sangat tinggi. Ia pun dikenal sebagai 6. Cermati kutipan teks biografi berikut ini!
peloporteks
Bacalah penyair angkatan
berikut untuk45. Dari esai-
menjawab soal no. 3 dan 4! Saat terjadi Agresi Militer II Belanda, Ibu kota Indonesis dipindahkan
esai dan sajaknya, jelas sekali ia seorang ke Yogjakarta karena Jakarta sudah diduduki oleh tentara Belanda.
individualis yang bebas. Dengan berani Soedirman memimpin pasukannya membela Yogjakarta dari
dan secara demonstratif, ia menentang serangan Belanda II tanggal 19 Desember 1948.
pula sensor Jepang dan ia selalu Makna kata diduduki pada teks biografi di atas adalah…
menjadi incaran kempetai (polisi rahasia
Jepang).
A. direbut C. diserang 8. Keistimewaan yang dapat dicontoh dari kehidupan tokoh tersebut
B. diambil D. dimiliki adalah ...
A. Mengutamakan kepentingan bangsa dalam berbuat sesuatu.
Bacalah teks berikut untuk menjawab soal no 7,8 dan 9! B. Berani menjabat ketua pemerintahan Darurat RI.
Syafruddin Prawiranegara C. Menjabat Menteri Keuangan pada Kabinet Syahrir.
Sebenar Ayah, di Saat Sempit dan D.Menerima jabatan Gubernur Bank Indonesia.
Lapang
Sepanjang perjalanan hidupnya,
Syafruddin Prawiranegara mendapati
banyak peristiwa penting menghampiri 9. Pesan yang tersirat dalam bacaan tersebut adalah ....
dirinya. Sebagiannya adalah pilihan- A. Jangan menganggap diri lebih pintar dari orang lain.
pilihan yang ia ambil dengan penuh B. Syukuri apa yang kita peroleh sebagai karunia Illahi.
keberanian. Sebagian lagi, adalah C. Jangan terlalu bangga dnegan apa yang kita dapat.
kesempatan-kesempatan yang D.Syukuri apa pun yang dilarang untuk kita.
memberinya ruang untuk berbuat
sesuatu, utamanya untuk bangsa ini di 10. Bacalah kutipan biografi tokoh berikut!
Rio Haryanto adalah pembalap yang menjanjikan untuk membawa
kala itu. Jabatan yang pernah
nama Indonesia ke dunia internasional. Dalam kejuaraan formula
didudukinya adalah Ketua
Asia 2.0 tahun 2008, ia menempati urutan pertama kategori Asia dan
Pemerintahan Darurat Republik
urutan ketiga kategori Internasional. Rio kini terjun ke kejuaraan
Indonesia (PDRI), Menteri Keuangan
balap mobil formula BMW Pacipic. Ia dinobatkan sebagai Duta
pada Kabinet syahrir, dan Gubernur
Nasional Antinarkoba pada peringatan Hari Antinarkoba.
Indonesia. Perjuangannya yang paing
Keistimewaan tokoh tersebut adalah …
terkesan adalah menerobos sampai A. Pembalap yang menjanjikan nama baik Indonesia
berjuang dari lebatnya hujan dalam B. Pembalap mobil Formula BMW Pacific
peristiwa yang dikenal dengan PRRI. C. Pembalap yang menempati urutan pertama pada tahun 2008
Tentu, di mata Syafrudin yang anak D.Pembalap yang dinobatkan sebagai duta nasional antinarkoba
7.seorang tokohdi atas
Kutipan biografi agama
termasuk Banten,
bagian … 11. Struktur teks pada kutipan biografi nomor 10 (Rio Haryanto)
kesemuanya
A. orientasi itu tetap dimaknai sebagi
karunia Illahi. termasuk bagian …
B. deskripsi A. orientasi C. peristiwa
C. peristiwa B. deskripsi D. reorientasi
D. reorientasi
Limbak dan Walter Spies menjadi sebuah seni pertunjukkan oleh
12. Bacalah kutipan biografi tokoh berikut! umum dan ditampilkan di berbagai negara di Eropa.
Sejak kecil, Dewi Sartika sudah menunjukkan bakat pendidik dan
kegigihan untuk meraih kemajuan. Hal ini didorong pula oleh 13. Berdasarkan struktur teks deskripsi, paragraf kesatu adalah ....
pamannya, Bupati Martanegara yang memiliki keinginan yang sama. A. definisi umum
Adat yang mengekang kaum wanita ada waktu itu membuat B. deskripsi bagian
pamannya mengalami kesulitan dan khawatir. Namun, karena C. deskripsi manfaat
kegigihan semangatnya, akhirnya Dewi Sartika bisa meyakinkan D. identifikasi
pamannya dan diizinkan mendirikan sekolah untuk perempuan.
14. Pernyataan yang tidak sesuai dengan teks di atas adalah … .
Hal yang patut diteladani pada tokoh tersebut adalah …
A. Tari kecak merupakan pertunjukan seni khas Bali.
A. Memiliki banyak bakat
B. Tarian ini sangat disukai wisatawan mancanegara.
B. Mudah khawatir saat mengalami kesulitan
C. Para penari itu mengenakan kain kotak-kotak seperti papan
C. Semangat yang gigih meraih cita-cita
D.Berusaha meyakinkan pamannya untuk maju. catur
D.Tari kecak pun menggambarkan kisah Ramayana.
Bacalah teks deskripsi berikut untuk menjawab soal nomor 13
dan 15!
15. Perhatikan kutipan suatu bagian teks deskripsi yang acak berikut!
Tari kecak merupakan pertunjukan seni khas Bali yang
1) Tari Gambyong adalah tarian untuk menyambut tamu atau
diciptakan tahun 1930-an. Tari itu dimainkan oleh puluhan lelaki yang
mengawali suatu resepsi perkawinan.
duduk melingkar. Pada saat menari, mereka menyerukan kata “cak” 2) Penari ini hidup pada zaman Sunan Paku Buwana IV di
dengan irama tertentu seraya mengangkat kedua lengannya. Para Surakarta.
penari itu mengenakan kain kotak-kotak seperti papan catur melingkar 3) Tarian ini dinamai sesuai dengan nama penari yang bernama
pinggang mereka. Gambyong.
Tarian ini merupakan gambaran kisah Ramayana tatkala barisan 4) Dia mahir dalam menari dan memiliki suara merdu sehingga
kera membantu Rama melawan Rahwana. Rama ingin membebaskan menjadi pujaan kaum muda pada zaman itu.
Shinta yang diculik oleh Rahwana. Tari kecak diciptakan pada tahun Kalimat-kalimat tersebut akan menjadi bagian teks deskripsi yang
1930-an oleh I Wayan Limbak yang bekerja sama dengan pelukis sistematik jika disusun dengan urutan….
Jerman Walter Spies. Pada awalnya, kedua seniman itu terpesona oleh A. 1)-2)-3)-4) C. 2)-3)-1)-4)
tarian dalam ritual Sanghyang. Ketika itu, para penari Sanghyang B. 1)-3)-2)-4) D. 3)-2)-4)-1)
menari dalam kondisi kemasukan ruh. Ritual Sanghyang sendiri
16. Struktur teks deskripsi adalah …
merupakan ritual masyarakat yang bersumber dari tradisi pra-Hindu A. definisi umum, definisi bagian, definisi manfaat
dengan tujuan menolak bala. Ritual ini kemudian diadopsi oleh Wayan B. tesis, argument, penegasan ulang
C. identifikasi, klasifikasi, deskripsi
D. orientasi, peristiwa, reorientasi

17. Pernyataan berikut yang termasuk ciri-ciri teks deskripsi adalah …


A. Bersifat menceritakan
B. Mengandung bukti dan kebenaran
C. Menggunakan contoh, fakta, gambar peta, dan angka
D. Menggambarkan objek sejelas-jelasnya dengan melibatkan kesan
indera.

18. Gua peninggalan penjajahan Jepang itu tidak terawat. Banyak


coretan di dinding gua mengganggu pemandangan mata. Di pintu
gua ditumbuhi banyak rumput dan ilalang. Sampah-sampah
berserakan di dasar gua. Rupanya pemerintah setempat kurang
memedulikan objek bersejarah tersebut.
Paragraf tersebut termasuk paragraf …
A. deskripsi orang
B. deskripsi waktu
C. deskripsi tempat
D. deskripsi objek

19. Rangkaian koral di Laut Karimun Jawa berbagai bentuk dan


beraneka ragam. Ada koral berbentuk bunga berwarna krem dan 20. Pemerintah Kabupaten Probolinggo menawarkan paket wisata
merah muda, di tengahnya ada bintik-bintik menyerupai benang erupsi Gunung Bromo. Penawaran ini bertujuan untuk
sari. Begitu elok koral-koral itu. Di sisi lain ada karang dengan meningkatkan kunjungan wisatawan sekaligus memberikan
bentuk tumbuhan kaktus bergerigi, bercorak biru, dan hijau. jaminan bahwa gunung setiinggi 2.329 m dari permukaan laut
Keindahan koral lebih lengkap dengan ikan-ikan kecil berwarna etrsebut aman dikunjungi meskipun berstatus siaga. Wisatawan
oranye di atasnya. Luar biasa indah seperti lukisan dengan warna dapat menikmati pemandangan eksotis Gunung Bromo dari jauh.
yang kontras. Tidak ketinggalan koral berbentuk jamur kuping Penawaran paket wisata ini termasuk kegiatan menikmati hasil
dengan warna coklat dan krem sungguh mirip jamur panen dan keindahan matahari terbit.
sesungguhnya. Teks tersebut membahas …
Kalimat utama teks deskripsi di atas terdapat pada kalimat … A. Keindahan Gunung Bromo
A. kesatu B. Penawaran paket wisata erupsi Gunung Bromo
B. kedua C. Peningkatan kunjungan wisatawan
C. ketiga D. Pemandangan eksotis Gunung Bromo
D. keempat
***

PEMANTAPAN 9

Unsur Intrinsik
Teks Cerpen
Pilihlah salah satu jawaban yang paling tepat!

1. P Pagi itu, Fawaz, Dafa, dan


Dzaky berangkat bersama ke sekolah.
Di jalan mereka bertemu dengan Alvin.
Dafa menanyakan tugas Bahasa
Indonesia kepada Alvin. Dengan
menepuk dadanya, Alvin berkata,
“Tentu sudah dong. Aku gitu loh.”
“Yang benar, Vin?” tanya Dzaky
“Aku memang sudah
mengerjakan tugas itu dan dijamin
nanti pasti nilaiku paling bagus di
kelas kita,” kata Alvin.
C. (2) dan (4)
D. (4) dan (6)

Watak tokoh Alvin dalah …


A. setia kawan C. jujur
B. sombong D. acuh 3.
Sudah dua tahun, Mbok Jah
tidak berkunjung ke rumah mantan
2.
Emak menyeringai senang, majikannya di kota. Meskipun sudah
memperlihatkan sederetan gigi yang berhenti, Mbok Jah tetap memelihara
warna putihnya tak sempurna lagi. hubungan baik dengan keluarga
(2) Ia senang melihat anak laki- Mulyono. Dua puluh tahun telah
lakinya makan dengan lahap. (3) Ia dilewatinya bekerja sebagai pembantu
seketika lupa bagaimana anak-anak rumah tangga di keluarga Mulyono.
penduduk dusun ini mlempar Benjo Buat orang yang sudah tua
dengan tomat busuk dan batu kerikil. seperti Mbok Jah, apalah yang
(4) Sementara orang-orang dewasa dikehendaki selain berteduh dan
melihat kejadian itu tanpa bereaksi berkumpul dengan keluarganya.
apapun selain tertawa. (5) Ia tampak Namun, apalah dayanya. Anak
tersenyum mengingat kejadian itu. (6) semata wayangnya sudah tidak ingat
Apa yang lucu dari melihat seorang dia lagi. Anaknya lebih senang tinggal
bocah laki-laki yang pasrah begitu di Surabaya dengan istri dan
saja ditawar bocah-bocah sebayanya, anaknya.
dilempar batu hingga mengakibatkan
luka memar dan berdarah dari Tindakan terpuji tokoh yang terdapat dalam penggalan cerita di atas
sekujur tubuhnya. adalah …
A. Mbok Jah bekerja di keluarga Mulyono di desa.
Bukti watak tokoh Emak mempunyai sifat sabar dalam cerita B. Mbok Jah telah berkumpul dengan anak dan cucunya.
tersebut ditandai dengan nomor … C. Mbok Jah ditinggalkan anaknya yang senang tinggal di Surabaya
A. (1) dan (3) D.Mbok Jah tetap memelihara hubungan baik dengan mantan
B. (3) dan (5) majikannya.
4. .,
Bibirku tak sanggup lagi untuk
berbicara sepatah kata pun. Langsung
kunonaktifkan HP-ku. Dan kulempar
jauh dariku. Aku benar-benar tidak
pernah menduga. Bahwa ia akan
pindah. Kabar ini sungguh Amanat kutipan cerita di atas adalah ….
membuatku kehilangan tulang-tulang A. Bekerjalah dengan arif dan bijaksana!
rangka yang menopang tubuhku. B. Jangan iri pada kesuksesan orang lain!
Badanku terasa sangat lemah dan C. Berbuatlah kebaikan kepada siapa pun dan di mana pun.
aku hanya bisa terduduk lemas D. Maafkanlah kesalahan orang lain agar hati tidak kesal!
sambil menggigit guling yang ada di
genggamanku. 6. Bacalah kutipan cerita berikut!

Latar kutipan cerpen tersebut adalah … Mengapa berat sekali


Mengapa berat sekali
A. tempat duduk memanggil Tante Tante Tina
Tina dengan
dengan
memanggil
sebutan “Mama”?
“Mama”? Padahal
Padahal Tante
Tante
B. kamar tidur sebutan
Tina, mama
mama yangyang baik.
baik. Hari
Hari ini
ini
C. tengah rumah Tina,
Sekar melihat satu lagi kasih
D. kursi beranda Sekar melihat satu lagi kasih
sayangTanteTanteTina.
Tina.Pasti
PastiTante
TanteTina
Tina
sayang
ingin sekali
sekali dipanggil
dipanggil “mama”,
“mama”,
ingin
Hati Dinda merasa sangat sebal tetapitidak
tidakpernah
pernahmemaksa.
memaksa.
tetapi
kepada Winda karena ulah Winda yang Sekar juga
juga sangat
sangat sayang
sayang
Sekar
sangat menjengkelkan. Namun, kepadaTanteTanteTina,
Tina,tetapi
tetapidia
diaselalu
selalu
5. kepada
hatinya juga selalu bertanya-tanya teringat almarhum mama
teringat almarhum mama
tentang mengapa tidak memaafkannya kandungnya tiap tiap kali
kali akanakan
kandungnya
saja dan mengapa harus terus sebal memanggil Tante Tina dengan
memanggil Tante Tina dengan
kepada teman sendiri? sebutan “mama”“mama” atau atau Sekar
Sekar
sebutan
menjadibimbang.
bimbang.
menjadi
Ketika Dinda mendengar Winda
tidak masuk sekolah karena sakit,
hatinya memutuskan untuk segera
menengok Winda dan menyudahi Konflik pada cerpen tersebut adalah …
kekesalannya kepada Winda serta
memaafkannya.
A. Sekar selalu ingat kepada mama kandungnya. modern lama
B. Sekar tidak mau memanggil “mama” kepada Tante Tina. C. Menggunakan Menggunakan
C. Sekar ingin memanggil Tante Tina dengan sebutan “mama” bahasa tak bahasa resmi
D. Sekar sayang kepada Tante Tina, tetapi berat memanggilnya resmi
“mama” D. Menggunakan Menggunakan
7. Bacalah kutipan kedua novel berikut! bahasa tak bahasa baku
Kutipan novel 1 baku
Lelaki miskin itu akhirnya pulang
dengan hati gembira. Tidak lupa ia 8. Bacalah kutipan cerpen berikut!
membeli baju dan makanan untuk Pada waktu istirahat aku memakan bekal
anak istrinya. Ia berharap semoga yang dikasih oleh Sasha. Rasanya enak banget.
tidak ada lagi manusia serakah Ketika aku hendak menyantap untuk yang kedua
seperti dua satpam itu. Mereka sudah kalinya, tiba-tiba Ziska menjatuhkan bekal itu dari
digaji, tapi masih serakah hendak bangku ke bajuku. Sayur yang hangat menumpahi
mengambil harta milik orang lain
bajuku lalu Ziska menginjak bekal dari Sasha.
secara jahat.
Kemudian Shasa pun datang."Ada apa ini?"
Kutipan novel 2
Open diminta di kompeitei “Lihat! Masak anak gembel kaya dia bisa
sebenarnya maksa Open tidak perlu bawa bekal enak seperti itu. Gak mungkin kan, ya
sama sekali karena ia tokoh akan udah aku buang aja daripada dia makan barang
mengatakan dengan terus terang haram.” Ujar Ziska ketus "Enak aja, bekal itu
Cara pengarang menggambarkan watak tokoh Sasha dalam
bahwa karangan itu memang pemberian dariku tau...! aku gak mau temenan
kutipan cerpen tersebut adalah....
dimaksudkan begitu. Tapi katanya sama kamu lagi...!!!"tokoh lain
A. penceritaan
pula dongeng itu bukan dibikin- B. dialog antartokoh
bikinnya begitu saja tapi betul-betul C. penggambaran fisik
pernah didengarnya. D.penyebutan langsung
Perbedaan penggunaan bahasa kedua novel tersebut adalah …
Kutipan Novel 1 Kutipan Novel 2 9. Cermati kutipan cerpen berikut ini!
A. Menggunakan Menggunakan “Good morning Mam. Selamat pagi,” kata seorang penjual di Pasar
ragam bahasa ragam bahasa Sukawati
baku bebas “Selamat pagi, ‘ jawab si pembeli. Dia seorang turis asing.
B. Menggunakan Menggunakan ‘Mari, mau beli apa?” “
bahasa sastra bahasa sastra Ada patung Garuda Wisnu Kencana yang terbuat dari kayu”
Pengubahan cerpen di atas menjadi bentuk monolog adalah … oleh kilat dan petir. kucuci setelah
A. Pada suatu pagi, seorang ibu muda dari Eropa pergi ke Pasar Dan adalah jalan itu mandi. Pakaianku
Seni Sukawati untuk membeli patung Garuda Wisnu Kencana sangat berbelok- kuambil dengan
yang terbuat dari kayu. Setelah sampai di Pasar Seni Sukawati, belok, sehingga tak hati-hati dari lemari
ia mendatangi sebuah toko, lalu sang pemilik toko menanyakan mudah ditempuh agar tetap rapi.
apa yang hendak sang ibu beli, lalu sang ibu menjawab bahwa ia serta turun naik Buku yang telah
ingin membeli patung Garuda Wisnu Kencana yang terbuat dari dengan susahnya. selesai kubaca,
kayu. dikembalikan ke
B. Seorang ibu muda dari Eropa datang ke Pasar Seni Sukawati. Ia tempatnya.
mendatangi sebuah toko kelontong, lalu pemilik toko Perbedaan karakteristik kedua kutipan novel tersebut adalah …
menanyakan apa yang hendak sang ibu beli, lalu sang ibu Kutipan 1 Kutipan 2
menjawab bahwa ia hendak membeli patung Garuda Wisnu A. Latarnya jelas Latarnya kurang
Kencana yang terbuat dari kayu. jelas
C. Seorang gadis manis dari Eropa pergi ke pasar seni Sukawati B. Bahasanya Bahasanya mudah
untuk membeli patung Garuda Wisnu Kencana dari kayu. berbelit-belit dipahami
Setelah sampai di Pasar Seni Sukawati, ia mendatangi sebuah Pengacara
C. Temanya muda itu berhenti
Temanyasebentar
toko kelontong dan alat tulis. untuk memberikan waktu
kebahagiaan pengacara
pendidikan
D.Seorang lelaki dari Eropa pergi ke Pasar Seni Sukawati untuk D. senior
Watak menyimak.
tokohnya Watak Kemudian ia
tokohnya
membeli patung Garuda Wisnu Kencana yang terbuat dari kayu. melanjutkan.
pemarah penyayang
Setelah sampai di pasar seni Sukawati, ia mendatangi sebuah “Tapi aku datang kemari bukan untuk
toko. Lalu,sang pemilik toko dengan wajah masam menanyakan 11. meminta
V pertimbanganmu, apakah
apa tujuan mereka datang ke rumah. keputusanku untuk menolak itu tepat
10. Bacalah kedua kutipan novel berikut! atau tidak. Aku datang kemari karena
Kutipan 1 Kutipan 2 setelah negara menerima baik
Akan tetapi, mereka Aku juga sudah penolakanku, dia sendiri datang ke
yang malang dan terbiasa sendiri tempat kediamanku dan meminta
celaka, tiada lain dan mengurusi dengan hormat supaya aku bersedia
yang diperolehnya diriku seorang diri. membelanya.”
di jaian itu, Aku belajar “Lalu kamu terima?” potong
melainkan panas menjadi orang yang pengacara tua itu tiba-tiba.
terik atau hujan rapi dan disiplin. Pengacara muda itu terkejut. Ia
lebat dan angin Baju yang telah menatap pengacara tua itu dengan
topan, yang disertai dipakai langsung heran.
“Bagaimana Anda tahu?”
12. Latar tempat kutipan cerpen di atas adalah …
A. rumah
B. ruang makan
C. kamar tidur
Penggambaran watak tokoh “pengacara muda” dalam kutipan cerpen D. sekolah
tersebut adalah ...
A. diceritakan tokoh lain 13. Jenis konflik yang terdapat dalam kutipan cerpen di atas adalah
B. diceritakan pengarang …
C. dialog antartokoh A. Ide
B. lahir
D.penggambaran perilaku
C. batin
D. pemikiran
Bacalah kutipan cerpen berikut untuk menjawab soal nomor 12
dan 13!
‘Aku sangat berterima kasih
karena kedua orangtuaku telah
membesarkanku. Namun, keringat
yang mereka keluarkan untuk Bacalah kutipan cerpen berikut untuk menjawab soal nomor 14
menyekolahkanku, memberiku makan dan 15!
dan memenuhi kebutuhanku hanya
menyisakan kesakitan batinku. Aku Aku memandangnya lama, tanpa
mengucapkan sepatah kata pun. Ini
tak punya hak atas diriku sendiri. Aku
ketiga kalinya ia berada di tempat ini.
hanya punya kewajiban yang harus
Melakukan hal yang tidak wajar. Ia
aku tunaikan. Sekolah di lembaga yang berbicara pada batu-batu! Ya, pada batu-
mereka inginkan, keluar rumah hanya batu. Ia bisa tampak serius, lalu tiba-tiba
ketika mereka mengizinkan, dan ia tertawa atau menangis sendiri. Ia
melakukan hal-hal yang membuat membelai batu-batu. Menggendongnya
mereka senang.’ seperti menggendong bayi, memasukkan
Notes hitam kembali menampung batu-batu tersebut ke dalam tas kainnya
derai air mata. Mengakhiri malam- yang kusam.
\ (Hingga Batu Bicara; Helvy Tiana Rosa)
malam dengan cara yang sama,
tertidur dengan mata basah dan notes
hitam di genggaman.
14. Tokoh utama dalam cuplikan cerita tersebut adalah… 17.
A. aku (1) Said menyiapkan tiga butir
B. batu telur dan mempersilakan temannya
C. bayi makan. (2) Abdullah mengambil telur
D. ia satu lalu mengupas kulitnya pelan-
pelan. (3) Sementara Said, mengupas
15. Sudut pandang yang terdapat dalam penggalan cerita di atas dengan cepat, lalu menyantapnya,
adalah … demikian juga dengan telur
a. orang pertama pelaku utama
ketiganya. (4) Setelah selesai
b. orang pertama pelaku sampingan
c. orang kedua pelaku sampingan mengupas telur, Abdullah mengambil
d. orang ketiga di luar cerita pisau dan membelahnya menjadi
dua. (5) Yang satu dipegangnya dan
16. yang satu diberikannya pada Said.
Iwan sungguh menakutkan bagi anak-
(6) Tak ayal lagi, Said menangis
anak yang suka mencuri mawar-
mawar ibu. Pagi-pagi, dia berjemur di terharu.
kebun, mandi dalam cahaya Bukti bahwa tokoh Abdullah “dermawan” terdapat pada kalimat
matahari. Dengan mata setengah
nomor …
terpejam, dia mengintai siapa-siapa
A. (6) B. (5) C. (4) D. (2)
yang mendekati kebun. Bila anak-
anak yang berani, menjulurkan
tangannya ke dalam kawat, Iwan lalu 18. Fitri bukan orang yang mudah kagum dan mudah heran terhadap
marah. Dan sebelum anak itu sesuatu. Keisyaratannya akan harga dirinya sangat besar. Ia tahu
mengetahuinya, Iwan sudah berada di banyak yang akan dikerjakan dan dicapainya. Segala sesuatu
dekatnya, memandang pencuri itu diukurnya dari kecakapannya sendiri. Oleh sebab itu, ia jarang
dengan mata berkilat-kilat. mendapat kegagalan.
Watak Fitri pada penggalan cerpen di atas adalah …
Penggambaran watak tokoh Iwan pada kutipan cerita di atas adalah
A. sombong C. pandai dan cakap

B. Baik hati D. Rendah diri
A. Dialog antartokoh
B. Tindakah tokoh
Bacalah kutipan cerita berikut untuk menjawab soal nomor 19
C. Gambaran fisik tokoh
s.d. 20!
D. Diceritakan tokoh lain.
Perasaanku sangat berdebar ketika
menggigil itu, misalnya. Aku ingin memanggilnya, dan menyelipkan
memasuki bilik ujung itu. Cerita Emak itu sulit dua buah uang logam ini ke dalam tangannya, dan menyuruhnya
dipercaya. Tapi, aku dan istriku harus segera pulang. Atau buat bocah enam tahunan tadi, yang dari
menghormatinya. Saat mataku menatap meja hidungnya sudah merembes cairan kental. Mestinya, dia duduk
makan itu, memang sebagian nasi dan lauk manis di rumah, sambil minum susu hangat, dan makan ubi goreng.
pauknya sudah berkurang. Kelihatannya memang Sekarang pasti ia sudah kena pilek.
ada orang yang telah menyantap hidangan itu. Simpulan yang dapat diambil berdasarkan kutipan cerpen di atas
Tapi, aku tak percaya begitu saja bahwa adalah ,…
yang menyantap makanan itu adalah mendiang A. tokoh aku seorang yang perhatian.
ayah. Ketika Emak menunaikan shalat Dzuhur di B. tokoh aku seorang yang perasa.
mushala tengah rumah kami, aku berusaha C. Tokoh aku seorang yang kaya.
menyelidik seisi bilik rumah kami, aku berusaha D.Tokoh aku seorang yang dermawan.
meyelidik seisi bilik ujung. Beberapa saat, kudengar
suara dengus kucing sedang memakan sesuatu. Aku
22. Rumah Abah masih berdiri gagah. Rumah kayu sederhana
menemukan jawabannya. Pastilah yang menyantap
seakan melambangkan sikap gagah dan tabah. Tetap berdiri, walau
hidangan Emak adalah kucing tetangga yang
selalu menyelinap rumah kami.
di samping kanan kirinya telah berubah. Rumah tangga yang
berganti pemilik. Rumah itu juga indah, bagi yang bisa
19. Bukti bahwa kutipan cerpen tersebut merupakan tahapan merasakannya. Di bagian bawah masih ada kolam ikan yang cantik
penyelesaian dalam alur adalah … karena ada hiasan pohon teratai warna padma, memebri kesan
A. tokoh aku tidak memepercayai emaknya. manakala melihat air kolam yang kecoklatan. Sesekali ada riakan
B. tokoh aku kaget saat mengetahui hidangan berkurang. kecil dari ikan-ikan peliharaan yang bisa menjadi penyambung
C. tokoh aku menyelidiki siapa yang makan thidangan di meja. kehidupan.
D.tokoh aku menemukan jawaban atas prasangkanya. (Becak Emak, Arswendo Atmowiloto)
Penggalan novel tersebut merupakan tahapan alur pada bagian …
20. Nilai moral dalam kutipan cerpen tersebut adalah … A. pendahuluan
B. klimaks
A. Seorang anak yang mencari bukti dengannya.
C. penanjakan laku
B. Seorang anak yang menghormati ibunya. D.antiklimaks
C. Seorang anak yang tidak percaya hal mistik
D.Seorang anak yang berusaha mencari kebenaran. Bacalah kutipan novel berikut untuk menjawab soal nomor 23 dan
24!
21. Aku jadi merasa tidak enak. Rasanya uang ini lebih berarti di Lintang memang tak memiliki pengalaman emosional dengan Bodenga
tangan mereka. Di tangan bocah yang berkaos biru, yang sedang seperti yang aku alami, tapi bukan baru sekali itu ia dihadang buaya
dalam perjalanan ke sekolah. Dapat dikatakan, tak jarang Lintang (2)Lalu dia menunggu hewan hutan yang akan melewatinya
mempertaruhkan nyawa demi menempuh pendidikan, namun tak sehari (3) Tidak lama kemudian beberapa domba gunung berjalan ke
pun ia pernah bolos. Delapan puluh kilometer ditempuhnya dengan arah dirinya
sepeda setiap hari. Tak pernah mengeluh, jika kegiatan sekolah (4) Kuda itu bergegas mencari tempat untuk bersembunyi
berlangsung sampai sore, ia akan tiba malam hari di rumahnya. Sering (5) Tempat itu harus terlihat gelap dan sering dilalui oleh
aku merasa ngeri membayangkan perjalanannya. beberapa hewan hutan
(Laskar Pelangi, Andrea Hirata) (6) Akhirnya dia menemukan semak-semak yang cukup gelap
23. Sudut pandang penggalan novel di atas adalah … untuk bersembunyi dan kuda itu pun masuk ke semak-semak
A. orang ketiga serba tahu dengan menggunakan kulit harimaunya.
B. orang ketiga pelaku sampingan Urutan kalimat-kalimat di atas agar menjadi teks fabel yang baik
C. orang pertama pelaku sampingan
adalah …
D. orang pertama pelaku utama
A. 1-2-3-4-5-6 C. 1-4-2-3-5-6
24. Amanat yang terdapat dalam penggalan novel di atas adalah … B. 1-4-2-3-5-6 D. 1-4-5-6-2-3
A. Jangan pernah menyerah dalam menempuh pendidikan.
B. Carilah ilmu sampai ke ujung dunia. 27. (1) Hampir setiap malam mereka berkumpul bersama, berpesta,
C. Jangan takut menghadapi binatang buas seperti buaya. menari, dan bergembira. Mereka saling berbagi makanan kecuali
D.Berhemat pergi sekolah dengan menggunakan sepeda.
seekor belalang yang selalu hidup menyendiri. Ia hanya memandang
keramaian dari depan rumahnya. (2) Tingkah belalang itu sangat
25. (1) Hampir setiap malam mereka berkumpul bersama, berpesta,
aneh, ia malu karena ia telah kehilangan sebuah kakinya. (3) Kakek
menari, dan bergembira. Mereka saling berbagi makanan kecuali
Cacing pernah bercerita, Paman Belalang setahun yang lalu telah
seekor belalang yang selalu hidup menyendiri. Ia hanya memandang
kehilangan kakinya akibat ia berkelahi dengan seekor burung yang
keramaian dari depan rumahnya. (2) Tingkah belalang itu sangat
hendak memangsanya. (4) Sehari-hari Paman Belalang hanya
aneh, ia malu karena ia telah kehilangan sebuah kakinya. (3) Kakek
duduk termenung meratapi kakinya yang hilang.
Cacing pernah bercerita, Paman Belalang setahun yang lalu telah Iwan sungguh menakutkan bagi anak-
Makna
anak penyesalan
yang sukadalam fabel mawar-
mencuri di atas terdapat pada kalimat nomor
kehilangan kakinya akibat ia berkelahi dengan seekor burung yang
…mawar ibu. Pagi-pagi, dia berjemur di
hendak memangsanya. (4) Sehari-hari Paman Belalang hanya
A. (1)
kebun, mandi B. (2) cahaya
dalam C. matahari.
(3) D. (4)
duduk termenung meratapi kakinya yang hilang.
Dengan mata setengah terpejam, dia
Makna penyesalan terdapat pada kalimat nomor …
mengintai siapa-siapa yang mendekati
A. (1) B. (2) C. (3) D. (4)
28. kebun. Bila anak-anak yang berani,
menjulurkan tangannya ke dalam
26. Cermatilah kalimat-kalimat berikut! kawat, Iwan lalu marah. Dan sebelum
(1) Lalu terlintas dalam benak kuda itu untuk menakut-nakuti anak itu mengetahuinya, Iwan sudah
hewan hutan lainnya berada di dekatnya, memandang
pencuri itu dengan mata berkilat-kilat.
Di sebuah hutan, hidup berbagai macam
hewan seperti kucing, kelinci, kerbau,
harimau, gajah, singa, zebra, dan lain-
lain. Mereka selalu hidup rukun dan
damai. Mereka saling membantu,
berbagi, dan saling menyayangi satu
sama fabel
Bacalah lain. berikut
Mereka untuk
hidupmenjawab
di dalamsoal nomor 30 dan 31!
ketenangan.
Pada suatu hari, mereka
memperdebatkan siapa yang pantas
menjadi Raja di hutan itu. Mereka
sedang dilanda rasa bingung. Tapi
dengan percaya diri yang tinggi gajah
Penggambaran watak tokoh Iwan pada kutipan cerita di atas adalah
mencalonkan diri sebagai Raja.
… “Aku tahu siapa yang sangat pantas
A. Dialog antartokoh menjadi Raja di hutan ini, yaitu … Aku!!
B. Tindakah tokoh Ha..ha..ha..!” seru Gajah dengan
C. Gambaran fisik tokoh lantang.
D. Diceritakan tokoh lain. “Ya, ya, usul yang bagus!” Beberapa
hewan seperti kelinci, zebra, jerapah,
29. Tak banyak tahu, apa kerjaan Mbah Tejo dan Mbah Marti di kerbau, sapi, dan burung merak
gubuk reotnya di pinggir Kalitambun, pekuburan yang sudah jadi menyetujui usul gajah.
lahan pekerjaan Mbah Tejo puluhan tahun. Setahu orang-orang, “Ah, tidak, tidak .. itu usul yang jelek.
Mbah Marti begitu setia pada Mbah Tejo. Sehari-hari mencari pucuk Yang sangat pantas menjadi Raja
singkong di sekitar pekuburan, atau dedaunan apapun untuk makan adalah Singa!!! Ha..ha..ha..” Seru Singa.
“Iya kami setuju!” kata ular cobra,
si Mbah.
anjing, dan haimau.
Kutipan cerpen di atas termasuk bagian …
“Aku yang jadi Raja!”
A. abstrak
“Harusnya itu aku!”
B. komplikasi “Tidak! Aku!”
C. orientasi “AKu!”
D.koda “Sudah! Lebih baik kita adakan
pemilihan saja! Seperti yang dilakukan
manusia sekarang. Kita sudah punya 2
calon. Siapa yang mendapat suara
terbanyak, ia yang akan menjadi Raja.”
Kelinci yang bijaksana menengahi.
AKhirnya, mereka sepakat untuk
mengadakan pemilihan esok harinya.
Washington DC, December 2003, jam 16.00
Iseng saja, aku mendekat ke jendela kaca dan
30. Simpulan
menyentuh pesan tersirat
permukaannya dengan dalam
ujung kutipan
telunjuk fable tersebut adalah …
A. Sifat
kananku. curang
Hawa dinginakan
segeramembuat kita celaka.
menjalari wajah dan
lengan kananku. Dari balik kerai tipis di lantai empatdan prasangka buruk
B. Jauhkan diri kita dari keserakahan
ini, C.
salju tampak
Sikap turun menggumpal-gumpal
percaya diri diperlukan jika seperti
ingin menjadi pemimpin.
kapas yang dituang
D.Sikap dari langit.
demokratis Ketukan-ketukan
diperlukan dalam memilih calon pemimpin.
halus terdengar setiap gumpal salju menyentuh kaca
di depanku.
31. Matahari
Struktur tekssore menggantung
yang tepat pada condong
fabelketersebut adalah ...
baratA. berbentuk
orientasipiring putih susu.
– resolusi – koda – komplikasi
Tidak jauh, tampak The Capitol, gedung
B. resolusi – koda – orientasi – komlikasi 32. Unsur intrinsik yang sangat menonjol dalam penggalan novel di
parlemen Amerika Serikat yang anggun putih
C. orientasi – komplikasi – resolusi – koda atas adalah …
gading, bergaya klasik dengan tonggak-tonggak
D.koda A. alur C. latar
besar. Kubah– resolusi
raksasanya – komplikasi – orientasi
yang berundak-undak
B. penokohan D. amanat
semakin memutih ditaburi salju, bagai mengenakan
Bacalah
kopiah haji. teks berikut
Di depan untuk
gedung menjawab
ini, hamparan soal nomor 32, 33, dan 34!
pohon
33. Sudut pandang yang terdapat dalam penggalan novel di atas
american elm yang biasanya rimbun kini tinggal
adalah ….
dahan-dahan tanpa daun yang dibalut serbuk es.
Sudah 3 jam salju turun. Tanah bagai dilingkupi A. Orang pertama pelaku utama
permadani putih. Jalan raya yang lebar-lebar mulai B. Orang pertama pelaku sampingan
dipadati mobil karyawan yang beringsut-ingsut C. Orang kedua pelaku utama
pulang. Berbaris seperti semut. Lampu rem yang D. Orang ketiga serba tahu
hidup-mati-hidup-mati memantul merah di salju.
Sirine polisi—atau ambulans-- sekali-sekali
menggertak diselingi bunyi klakson. 34. Latar penggalan novel di atas adalah …
(Negeri 5 Menara; A.Fuadi)
A. sore hari di kantor langit, dan … rembulan yang kini merah
B. dini hari di rumah jambu dalam pandanganku!…
C. malam hari di taman “Cerpen Rembulan di Mata Ibu;
D.sore hari di The Capitol Asma Nadia”
Struktur teks yang membangun kedua teks di atas adalah …
35. Cermatilah kutipan kedua teks berikut! Fabel Cerpen
A. Koda resolusi

PEMANTAPAN 10
Teks fabel
Dikisahkan pada suatu hari yang cerah B. komplikasi orientasi
ada seekor semut berjalan-jalan di C. resolusi komplikasi
D. orientasi koda
taman. Ia sangat bahagia karena bias
berjalan-jalan melihat taman yang
indah. Sang semut berkeliling taman ***
sambil menyapa binatang-binatang yang
berada di taman itu.
Teks cerpen

Siap UN Bahasa
Ibu menunjuk purnama yang
benderang. Aku mengikuti telunjuknya.
Batinku terasa lebih segar. Rembulan
merah jambu … itukah yang di inginkan
Ibu, menjadi seseorang. Menjadi orang
dalam arti sebenarnya. Punya karakter
dan prinsip yang berbeda. Siap
Indonesia 2019
mengarungi kerasnya selama ini? Hatiku
berbunga-bunga. Semua kehampaan,
kebencian, dan kekesalanku pada
wanita tua itu tiba-tiba terbang ke awan.
Aku tak lagi membencinya! Ternyata aku
cukup punya arti dimata Ibu. Aku
rembulan di hatinya! Tanpa ragu,
kepeluk Ibu erat. Bersama-sama, kami
menghabiskan waktu yang tak
terlupakan di beranda memandangi
B. Sederet film asing berbicara tentang lingkungan hidup.
N.H.
C. Film-film H.Dini,
Dini,
dalam lahir  29 Februari
peringatanFebruari
Hari Bumi Sedunia mengupas
N. lahir  29 
1936
Kondisi dan di   Semarang.
1936 di  pelestari Semarang.
bumi.
D.Berpendidikan 
Para percinta bumi SMA  bagian 
Berpendidikan  SMA gigihbagian 
merawat lingkungan masing-masing.
Sastra   (1956),  Kursus Pramugari
Sastra   (1956),  Kursus Pramugari
3. Cermatilah
Darat GIA GIAkalimat-kalimat
Jakarta (1956), berikut….
(1956), dan 
Darat Jakarta dan 
1) Kursus
KelompokB-1 kamiJurusan 
memutuskan Sejarahuntuk mengamati green house
Kursus B-1 Jurusan  Sejarah
sekolah,Tahun 1957-1960   bekerja
(1957).
(1957). Tahun 1957-1960   bekerja
2) di 
Tanaman-tanaman
GIA Kemayoran  tersebut, antara lain sirih merah, serai,
Jakarta.
di  GIA Kemayoran  Jakarta.
Pilihlah salah satu jawaban yang paling tepat! kemangi,rnenikah, 
Setelah jambu biji,berturut-turut
dan sirsak.
Setelah rnenikah,  berturut-turut
3) bermukim
Pagi tadi, kamiJepang, ditugaskan oleh guru bahasa Indonesia untuk
bermukim didi Jepang, Kamboja, 
Kamboja, 
1. Cermatilah teks ulasan berikut! melakukan
Filipina, pengamatan
Prancis. Amerikadi lingkungan sekolah.
Serikat,
Filipina, Prancis. Amerika Serikat,
Dalam rangka memperingati Hari 4) dan
Sungguh pembelajaran
sejak1980 1980 menetap 
menetap  yang menyenangkan hari itu.
kembali
dan sejak kembali
Bumi Sedunia, sederet film asing 5) di
Hasil identifikasi
Indonesia tersebut kami presentasikan di depan
(Jakarta-Semarang).
di Indonesia (Jakarta-Semarang).
mengenai lingkungan hidup kelompokN. H. H.lainDini  mulai 
mulai  menulis 
menulis 
N. Dini 
diputar. Film-film tersebut 6) tahun 
Di green house,
1951, kami mengidentifikasi
kemudian   sajaknya sejumlah tanaman yang
tahun  1951, kemudian   sajaknya
bisa dijadikan
dibacakan    obatdan    dimuat  
didiRRI
mengupas berbagai kasus dan RRI
dibacakan    dan    dimuat  
kondisi bumi saat ini serta orang Urutan
di kalimat
Majalah  tersebut
Budaya  danagar menjadi teks laporan yang tepat
Gadjah
di Majalah  Budaya  dan Gadjah
orang pelestari bumi yang penuh adalah
Mada,  ....keduanya   di  Yogyakarta.  
Mada,  keduanya   di  Yogyakarta.  
inspirasi. Sebagian besar film A.Tahun
(1) - (2) - (5) -cerpennya  
1953, (6) - (3) - (4)dimuat  
Tahun 1953, cerpennya   dimuat  
tersebut menceritakan sosok yang B.di (1) - Majalah 
(3) - (6) - (4)Kisah, 
- (2) - (5) Mimbar
di  Majalah  Kisah,  Mimbar
mengabdi secara total kepada alam. C.Indonesia,
(3) - (1) - (2)dan  - (4) -Siasat. 
(6) - (5) Tahun-
Indonesia, dan  Siasat.  Tahun-
Mereka gigih merawat kondisi bumi D.tahun
(3) - (1) - (6) - (2) - (5)
selanjutnya, - (4)
cerpennya pun
tahun selanjutnya, cerpennya pun
di lingkungan masing-masing. dimuat  dalam dalamberbagai  
berbagai  media.
media.
dimuat  
Cermatilah Tahun  kutipan 1955, biografiN.H. Dini berikut dengan saksama
Ide pokok teks ulasan tersebut adalah ... Tahun  1955, N.H. Dini
untuk mengerjakan hadiah 
memenangkan   soal nomor 4-6!
pertama
A. Pemutaran film dalam rangka memperingati Hari Bumi. memenangkan   hadiah  pertama
B. Film yang mengupas proses pelestarian lingkungan. penulisan   skenario  
skenario   sandiwara 
sandiwara 
penulisan  
C. Film yang menceritakan pengabdian seorang pelestari bumi radio se-Jawa 
se-Jawa  Tengah.Tengah. Tahun  Tahun  
radio
D.Cara melestarikan lingkungan yang baik 1987,Dini Dinimemenangkan  
memenangkan  hadiah hadiah
1987,
pertama  (untuk  peserta
2. Simpulan isi teks ulasan tersebut adalah ... pertama  (untuk  peserta
lndonesia)  lomba   lomba   mengarang   
mengarang   
A. Film-film asing yang diputar dalam rangka lndonesia) 
cerita  dalamdalam bahasa  
bahasa   Prancis 
Prancis 
memperingati Hari Bumi Sedunia. cerita 
yang  diselenggarakan  
diselenggarakan   Le Le Monde
Monde
yang 
danRadio 
Radio Franve 
Franve internasionale.
internasionale.
dan
A. N.H. Dini lahir di Semarang. Setelah menikah tinggal di
beberapa negara. Dia mulai menulis tahun 1951 dan
mendapatkan berbagai penghargaan atas karyanya sebagai
juara pertama.
B. N.H. Dini lahir di Semarang. Setelah lulus kuliah, dia menikah
sampai akhimya tinggal di Semarang. Setelah mulai
menulis tahun 1951 dia mendapatkan penghargaan.
C. Setelah lulus kuliah N.H. Dini menikah lalu tinggal di luar
negeri. Dia mulai menulis tahun 1951 dan memenangkan
beberapa penghargaan atas karyanya seperti juara pertama
lomba skenario sandiwara radio dan Lomba Mengarang Cerita
dalam bahasa Prancis.
D. N.H. Dini yang lahir di Semarang menjadi pramugari dan
tinggal di beberapa negara setelah menikah sebelum menetap
di Semarang. Dia mulai menulis tahun 1951 dan
mendapatkan berbagai penghargaan atas karyanya.

5. Arti kata skenario dalam teks biografi tersebut adalah ....


A. teks dialog drama yang dibacakan saat bermain di
panggung
B. tulisan berisi rencana lakon sandiwara atau film berupa
dialog
C. teks cerita drama atau film yang diikutkan lomba
D. tulisan cerita drama atau film yang digunakan saat syuting

6. Paragraf pertama teks biografi tersebut merupakan bagian ….


A. orientasi
B. peristiwa
C. masalah
D. reorientasi
4. Ringkasan isi teks biografi tersebut adalah ...
7. Cermatilah kutipan ulasan berikut!
Oleh karena itu, film Laskar
Pelangi layak disebut sebagai salah
satu film terbaik Indonesia sepanjang
masa. Meskipun film ini diproduksi pada
tahun 2008, Laskar Pelangi masih
layak untuk
Olahragaditontonsepeda
kapan pungunung
secara
berulang-ulang.
membutuhkan kemampuan Film ini juga fisikdapat
dan
ditonton oleh semua
mental karena adanya umur karena alam.
tantangan tidak
ada satu pun
Olahraga ini adegan kekeresanaspek
juga memiliki atau
sensual. Diharapkan
rekreasi. Karena lokasinya di berada
waktudi
mendatang semakin berbatu
daerah pegunungan banyak produser
dengan
film lndonesia yang
pemandangan alamtermotivasi
yang eksotis untukdi
membuat
ketinggian.film Anda
denganbisakualitas yang
menikmati
sama,
suasana bahkan melebihi film Laskar
yang tenang. Ringkasan yang tepat sesuai dengan isi teks tersebut adalah ...
Pelangi.Bersepeda di lereng-lereng A. Sepeda gunung tidak hanya untuk berolahraga, tetapi juga
gunung juga bisa dilakukan bersama berwisata karena dapat menikmati pemandangan yang eksotis di
beberapa
Kutipan ulasanorang.filmKetika
tersebutberhenti
merupakandi bagian .... ketinggian.
ketinggian
A. orientasitebing gunung, Anda dapat
C. evaluasi B. Kegiatan sepeda gunung membutuhkan kemampuan fisik dan
berteriak
B. tafsirankeras, seolah-olah terlepas
D. rangkuman mental. Selain untuk berekreasi, sepeda gunung juga bagus
dari semua masalah. Anda pun akan untuk kesehatan dan membuat tubuh menjadi lebih bugar.
Cermatilah
8. merasa teks eksposisi
excited dan stresberikut! yang C. Bersepeda di gunung dapat meningkatkan kemampuan dan
membebani pikiran akan hilang. kekuatan anggota tubuh Selain itu olah raga sepeda gunung
Bahkan sesekali Anda bisa juga tergolong kegiatan yang murah.
menaklukkan liku-liku jalan berbatu D.Wisata sepeda gunung mulai dikenal di Indonesia pada dekade
pegunungan, pasti akan ketagihan. '90-an.
Seperti olahraga lain, sepeda
gunung bagus dari segi kesehatan. 9. Cermatilah petunjuk pembuatan cincau lidah buaya berikut,
Bersepeda di gunung dapat 1) Siapkan irisan daging daun lidah buaya yang telah dicuci bersih
meningkatkan kemampuan atau secukupnya.
kapasitas jantung dan paru-paru, 2) Masukkan irisan daging daun lidah buaya dalam santan
kekuatan dan daya tahan otot.
Tubuh pun terasa lebih bugar.
3) Siapkan air santan kelapa diberi gula aren secukupnya dalam
panci Kalimat yang tepat untuk melengkapi bagian rumpang pada
4) Tuanglah hasil rebusan dalam mangkuk atau gelas. teks tersebut adalah ...
5) Rebuslah daging lidah buaya dan santan tersebut sampai A. Kira harus segera menanggulangi banjir.
mendidih. B. Alam jadi tidak memiliki resapan air lagi
6) Cincau daging daun lidah buaya siap disajikan C. Tanggung jawab siapakah alam yang rusak itu?
Urutan petunjuk pembuatan cincau lidah buaya agar menjadi D. Alam kita sudah direboisasi dengan tanaman.
teks prosedur yang tepat adalah ….
A. (1) - (2) - (3) - (4) - (5) - (6) 11. Cermatilah teks laporan observasi berikut!
B. (1) - (3) - (2) - (4) - (5) - (6) Selokan di Perumahan Cendana
C. (1) - (3) - (2) - (5) - (4) - (6) terpelihara dengan baik dan tidak
D. (1) - (3) - (2) - (6) - (4) - (5) mampat/tersumbat. Tempat pembuangan
sampah pun sudah ada di depan rumah
setiap warga. Tempat pembuangan sampah
tersebut berupa tong besar atau drum.
Kondisinya juga terawat sehingga tidak
menimbulkan bau tidak sedap. Petugas
kebersihan mengambil sampah setiap tiga
hari sekali. Selain itu, setiap satu bulan
sekali warga Perumahan Cendana
melakukan kerja bakti membersihkan
selokan doa rumput-rumput liar di sekitar
10. Cermatilah teks eksposisi berikut! perumahan.
Informasi yang tersurat dalam teks tersebut adalah …
Pada musim penghujan, kita selalu A. Lingkungan di Perumahan Cendana terpelihara dengan baik
disuguhi berita tentang banjir di B. Petugas kebersihan rajin mengambili sampah di depan rumah
warga.
Tanah longsor selama ini terjadi
mana-mana. Banjir melanda di berbagai
C. Di
karena depan
dua rumah setiap warga,
hal pertama terdapat tong atau drum sampah
karena
pulau di lndonesia. tidak hanya di
yang terawat.
Pulau Jawa, tetapi di luar Jawa juga air alam atau lingkungan dan kedua
D. Warga Perumahan Cendana suka bergotong-royong
turut meluap. Di samping karena faktor karena manusia. Kebiasaan manusia
membersrihkan lingkungan
alam, faktor manusia juga sangat yang menjadi penyebab utama
12. Cermatilah teks eksposisi berikut!
berperan sebagai penyebab teriadinya tanah longsor adalah
terjadinya banjir. Alam rusak kerena kerusakan alam, yaitu tidak
adanya penebangan hutan secara liar. melestarikan penghijauan lingkungan,
[…] terutama di [ ... ] perbukitan atau
pegunungan.
14. Cermatilah teks laporan berikut!
pada sabtu dini hari bupati
pandeglang, endang sukayana
mendatangi lokasi bencana bersama
kapolres AKBP fahmi idris dan
kepala badan penanggulangan
bencana daereh Pandeglang.

Kata yang tepat untuk melengkapi teks rumpang tersebut adalah


.... Perbaikan tanda baca yang tepat dalam paragraf tersebut
A. lokasi C. tempat adalah ...
B. kawasan D. tanah A. Pada Sabtu dini hari bupati Pandeglang, Endang Sukayana,
mendatangi lokasi bencana bersama Karpolres AKBP Fahmi
13. Cermatilah teks berikut! Idris dan Kepala Badan Penanggulangan Bencana
Para Bapak-bapak dan lbu-lbu Daerah Pandeglang.
sekalian. perpustakaan merupakan B. pada Sabtu dini hari Bupati Pandeglang, Endang Sukayana.
salah satu sumber belajar. Melalui mendatangi lokasi bencana bersama kapolres AKBP Fahmi
buku-buku koleksi di perpustakaan, Idris dan kepala Badan Penanggulangan Bencana
pengunjung dapar memperoleh ilmu Daerah Pandeglang.
pengetahuan dan mempelajari berbagai C. Pada Sabtu dini hari, Bupati Pandeglang, Endang
hal. Sukayana. mendatangi lokasi bencana bersama Kapolres
AKBP Fahmi ldris dan Kepala Badan Penanggulangan
Perbaikan kalimat pertama dalam teks tersebut adalah …. Bencana Daerah Pandeglang.
A. Para Bapak dan lbu, perpustakaan merupakan salah satu D. Pada Sabtu dini hari Bupati Pandeglang, Endang
sumber belajar. Sukayana, mendatangi lokasi bencana bersema Kapolres AKBP
B. Para Bapak-bapak dan lbu-ibu, perpustakaan merupakan Fahmi ldris, dan Kepala Badan Penanggulangan
salah saru sumber belajar. KetuaBencana
OSIS mengkoordinir pelaksanaan
daerah Pandeglang.
C. Bapak-bapak dan lbu-ibu sekalian, perpustakaan adalah lomba-lomba 17-an di sekolah. Kegiatan
salah saru sumber belajar. tersebut
15. berlangsung
Cermatliah teksdari laporan
jam 07.00 berikut!
s.d. jam
D. Para hadirin, perpustakaan merupakan salah satu sumber 14.00. Dalam kegiatan itu, diadakan pula
belajar. peragaan busana pahlawan oleh para siswa.
Para “peragawan dan peragawati” tampil
cukup mempesona.
Sudah sebulan kabut asap akibat
kebakaran atau pembakaran lahan
hutan dan lahan perkebunan melanda
Provinsi Riau dan Jambi. Berita di
media pemah melaporkan 80% udara
Sumatera diselimuti kabut asap,
penyakit ISPA menjadi langganan
masyarakat Riau dan Jambi, serta
beberapa lokasi di luar provinsi itu
terkena
Perbaikandampak asap kebakaran
kata bercetak atau teks tersebut adalah ...
miring dalam
pembakaran lahan, seperti
A. mengkoordinatif, pukul dimemesona
gunung
Sitoli, Pulau Nias.
B. mengoordinasi, pukul, memesona
Penerbangan
C. mengkoordinasi, dari
pukul, dan ke
mempesona
Pekanbaru. Jambi, pukul,
D. mengkordinatif, Kuala terpesona
Namu,
bahkan Palembang terganggu. Banyak
penerbangan dibatalkan. Demikian
pula di Kabupaten Mandailing Natal
dan Tapanuli Selatan, kabut asap juga
melanda bagian barat Provinsi Sumatera
Utara tersebut.
16. Cermatilah
Penumpang kutipan teks
pesawat tanggapan
terbang rute kritis berikut!
Padang Sidempuan dan Sibolga menuju
Kuala Namu terpaksa berpindah ke
moda angkutan darat yang ditempuh
dalam waktu 12 jam menuju pantai
timur Sumatera Utara.
Jarak pandang hanya 50 meter
dan udara bersih yang dihirup tinggal lsi yang ingin disampaikan penulis dalam teks tersebut adalah
5%. Penduduk Riau terancam ....
kesehatan paru-parunya pada waktu A. dampak negatif yang disebabkan oleh kabut asap di Provinsi
yang akan datang. Kekhawatiran warga Riau dan Jambi
Riau dan Jambi akhimya naik level B. pesawat yang sulit mendarat karena kabut asap
menjadi kemarahan karena tak pemah C. penduduk Riau, Jambi, dan sekitamya yang menderita akibat
tuntasnya pemberantasan kebakaran kabut asap
atau pembakaran lahan perkebunan
dan hutan di kedua provinsi tersebut.
D. penyakit yang mengancam penduduk akibat kabut asap Ketika anjing itu melewati pohon tersebut, sang gagak bertanya
kepadanya ''Ternyata kelinci itu lebih kencang dibandingkan dengan
Cermatilah kutipan teks fabel berikut dengan saksama untuk dirimu" kemudian sang anjing berkata dengan tenang, "Apa kau tidak
mengerjakan soal nomor 17-20! melihat perbedaan yang begitu mencolok antara aku dengan
Pada suatu pagi, anjing itu, bangun dari tidumya. kemudian dia kelinci itu?'' Sang gagak menjawab "Aku tidak melihat perbedaan
berjalan mengitari perkebunan jagung sambil mengendus-ngendus itu. Memang, apa perbedaan yang kau maksudkan itu?" Sang anjing
bau hewan lain dengan hidungnya. Penciuman anjing itu sangat tajam, menjawab, "Aku berlari untuk menangkap makanan, sedangkan dia
bahkan anjing itu mampu mencium bau kelinci dari jarak yang berlari mempertahankan hidupnya, sebuah keinginan akan
sangat jauh. menentukan kerasnya sebuah usaha"
Ketika anjing itu berjalan, ia mencium bau kelinci dari
kejauhan dan mengikuti arah bau itu Sampai akhirnya, dia melihat 17. Sebab konflik dalam cerita tersebut adalah ...
seekor kelinci sedang asik memakan pucuk jagung yang masih A. Anjing yang ingin memburu kelinci.
muda. Anjing itu berjalan perlahan mendekati kelinci tersebut. Ketika B. Kelinci yang suka memakan pucuk jagung yang masih
dia sudah sangat dekat dengan kelinci itu, sang anjing langsung muda.
mengejarnya dengan sangat cepat. Namun, sang kelinci mendengar C. Pak tani yang marah kepada kelinci.
langkah anjing itu karena kelinci memiliki telinga yang panjang dan D. Pertanyaan burung gagak kepada anjing.
sangat peka terhadap suara. Kelinci itu menghindari sang anjing
dengan cepat. Dia melompat dengan sangat cepat dan lompatan kelinci 18. Makna tersurat dalam teks fabel tersebut adalah ….
itu sangat jauh. A. Nasihat burung gagak yang bijak.
Sang anjing terus mengejarnya meskipun kelinci itu semakin B. Anjing yang pantang menyerah mengejar kelinci sebagai
menjauh dari jarak sang anjing. Namun, sang anjing tidak menyerah buruannya.
begitu saja. Anjing itu memiliki kemampuan berlari tanpa henti C. Kegigihan anjing mengejar kelinci karena ulah kelinci
sehingga dia mampu mengejar sang kelinci tanpa kelelahan. Meskipun yang suka memakan pucuk jagung yang masih muda.
demikian, sang kelinci yang sangat cepat melompat menghindari D. Kelinci yang suka memakan pucuk jagung yang masih
kejaran anjing itu membuat anjing itu kehilangan jejaknya. Anjing muda.
itu mulai mengendus-ngendus bau sang kelinci dan tidak lama
kemudian dia menemukan kelinci itu. Kini, dia mengejarnya lebih cepat 19. Pesan tersirat dalam kutipan teks fabel tersebut adalah ...
dari sebelumnya, tetapi sang kelinci itu tidak dapat dia kejar. Hingga A. Orang yang pantang menyerah pasti akan berhasil.
akhirnya anjing itu menyerah dan tidak melakukan pengejaran B. Berpikirlah sebelum melakukan perbuatan yang merugikan
terhadap kelinci itu lagi. orang lain.
Ternyata, kejadian itu ditonton oleh seekor burung gagak yang C. Nasihatilah teman yang berbuat curang atau tidak baik.
sedang bertengger di sebuah pohon yang daunnya sedang gugur.
D. Jika kita memiliki keinginan dan semangat yang kuat B. Nutrisi tersebut antara lain protein, lemak, serat, zat besi,
untuk mewujudkan apa yang kita inginkan, cepat atau lambat dan vitamin E.
keinginan itu pasti akan terwujud. C. Pemanfaatan jarak sebagai sumber energi altematif bukanlah
hal baru.
D. Indonesia berpotensi mengembangkan bahan bakar altematif
karena kekayaan alamnya.

20. Arti kata mengitari dalam teks fabel tersebut adalah …


A. mengelilingi
B. berhadap-hadapan
C. memutar balik
D. menuju ke arah

21. Kalimat yang tepat sebagai bagian definisi umum teks laporan
hasil observasi adalah ... 23. Cermatilah kalimat-kalimat berikut.
A. Kucing persia memiIiki ciri tubuh Iebih besar dan bulu yang 1) Wayang jemblung biasanya dipentaskan dalam acara hajat
lebih lebat serta indah. Oleh sebab itu, kucing jenis ini keluarga, seperti pesta perkawinan, khitan, atau acara tradisi,
jauh lebih mahal daripada kucing lokal. seperti sedekah bumi. dan lain-lain.
B. Biota laut adalah seluruh makhluk hidup yang 2) Wayang jemblung dulu banyak dijumpai di wilayah Kedu,
berkembang biak di laut. Biota laut itu antara lain Jawa Tengah.
terumbu karang, ikan, dan tumbuhan laut yang menjadi 3) Wayang jemblung, baik ceritanya maupun anak wayangnya
ekosistem laut. mirip dengan wayang kulit, tetapi semua iringan gamelan tidak
C. Air hujan yang terserap oleh cahaya matahari berubah menjadi dari peralatan karawitan yang ditabuh.
mendung. Udara dingin akan membentuk kristal-kristal air 4) Bebunyian iringan gamelan semua berasal dari mulut sang
yang memberat dan akhimya jatuh menjadi huian. dalang.
D. Banyak pengelola kantin sekolah yang kurang Urutan yang tepat dari teks laporan obsevasi tersebut adalah ....
memperhatikan kesehatan makanan yang dijual. Oleh sebab A. (1), (2), (3), (4)
itu, pendampingan oleh Dinas Kesehatan menjadi penting. B. (1), (3), (2), (4)
C. (2), (1), (3), (4)
22. Kalimat yang merupakan karakteristik teks eksposisi adalah ... D. (2), (3), (1), (4)
A. SMP Karya Mulia mengadakan program pemberantasan
narkoba.
Cermatilah kutipan teks berikut dengan saksama untuk C. Banyak para pelajar yang tidak menyadari pengaruh negatif
mengerjakan soal nomor 24-25. facebook.
Lingkungan
D. Hendaknyaadalah para
sesuatupelajar
yang ada mengendalikan
di diri dalam
sekitar manusia yang
menggunakan facebook. memengaruhi
Namun,  bagi  saya facebook
perkembangan kehidupan manusia,
lebih banyak merugikan.
baik langsung maupun tidak langsung.
Bagaimana  tidak? Sebagai seorang 
Bacalah
Komponentekslingkungan
berikut untuk
dapatmenjawab soal nomor 26-28!
dibedakan
pelajar, yang tentu  mempunyai  
menjadi lingkungan abiotik, biotik
kewajiban   belajar,  facebook sangat
social, dan budaya. LIngkungan abiotic
mengganggu  terhadap pelajaran 
adalah unsur lingkungan hidup yang
saya, baik dari segi  waktu maupun 
terdiri atas benda-benda tidka hidup,
konsentrasi   untuk  belajar.
seperti tanah, batuan, udara, dan lain-
Hampir  setiap hari orang-
lain. Lingkungan biotik adalah
orang membuka facebook hanya 
lingkungan hidup yang terdiri atas
sekadar untuk update status,  
makhluk hidup, seperti manusia,
chating,  dan banyak lagi.  Itu
tumbuhan, hewan, dan jasad renik.
hanya  mernbuang  buang  waktu,
Lingkungan social adalah lingkungan
padahal masih banyak  yang   lebih 
yang dibuat oleh manusia yang
penting  yang  harus  dikerjakan.
merupakan system nilai, gagasan, dan
keyakinan dalam perilaku sebagai
24. Kutipan teks eksposisi tersebut merupakan bagian …. makhluk hidup.
A. pemyataan pendapat Aktivitas manusiadalam memenuhi
B. tesis kebutuhan hidupnya dapat
C. argumentasi menimbulkan dampak terhadap
D. penegasan kembali lingkungan. Dampak terhadap
lingkungan muncul karena manusia
25. Kalimat yang tepat untuk melengkapi teks eksposisi tersebut cenderung mengambil sumber daya
adalah ... alam dari lingkungan secara berlebihan
A. Namun, keberadaan facebook saat ini seakan mampu tidak lagi sekadar mempertahankan
menyihir para penggunanya. kebutuhan hidup. Akibat dari perilaku
B. Para orang tua harus mengontrol pemakaian intemet anak- tersebut lingkungan mengalami
anaknya. kerusakan. Kerusakan lingkungan
makin parah seiring dengan
perkembangan kehidupan manusia.
26. Pertanyaan yang jawabannya sesuai dengan isi paragraph pertama
adalah …
A. Ada berapa jenis komponen lingkungan?
B. Apa saja manfaat lingkungan bagi manusia?
C. Bagaimana lingkungan mempengaruhi manusia?
D.Mengapa lingkungan terus mengalami kerusakan?

27. Makna kata ‘dampak’ dalam teks tersebut adalah … Bacalah kutipan teks cerpen berikut, kemudian kerjakan soal
A. persoalan nomor 29-31!
B. pengaruh
(1) Berkali-kali aku memohon Ibu
C. penyebab
menjemput Ena dan membawanya kembali
D.permasalah
tinggal bersama. Aku khawatir ia disiksa
anak-anak Paman. (2) Lagi pula, pamanku
28. Pernyataan yang sesuai dengan paragraf kedua tersebut adalah …
A. Lingkungan abiotic adalah jarang di rumah. Bibi sudah pasti membela
anak-anaknya. Ibu malah tertawa keras. (3)
unsur lingkungan hidup yang terdiri atas benda-benda tidak
Ketika aku mengulangi lagi permohonanku,
hidup.
Ibu memarahiku. “Saya tidak sanggup
B. Lingkungan adalah sesuatu
menanggung makan kalian berdua,”
yang ada di sekitar manusia yang mempunyai kehidupan
jawabnya, ketus. (4) Seandainya aku sudah
manusia baik langsung maupun tidak langsung. bekerja.
C. Dampak terhadap
lingkungan muncul karena manusia cenderung mengambil Ibu juga menganggur. Biaya hidup
sumber daya alam dari lingkungan secara berlebihan. kami ditanggung adik nenekku. Ibuku
29. Bukti wakak tokoh Aku perhatian terdapat pada nomor ….
D. Lingkungan sosial adalah mempunyai
A. (1) ibu yang kupanggil
C. (3) nenek dan
lingkungan yang dibuat oleh manusia yang merupakan system nenek
B. (2)mempunyai adikD. perempuan
(4) yang
kupanggil nenek juga. Nenek inilah yang
nilai, gagasan, dan keyakinan dalam perilaku sebagai makhluk
membantu
30. Konflik yang kami. Fatma
dialami adalahaku
oleh tokoh salah
yang tepat adalah …
hidup.
seorang pembantu
A. Anak-anak di rumah
paman yang sukanenek. Ibu
menyiksa
memperoleh banyak perlakuan
B. Ingin menolong Ena tapi istimewa
tidak adadari
biaya
C. Ibuterlebih
nenek, memerahi aku
setelah ia karena tidak
kabur dari punya uang.
rumah.
D.Ibu mendapat perlakuan istimewa setelah kabur.
merupakan hasil dari pembiasan cahaya. (4) Pelangi akan tampak
31. Amanat yang sesuai dengan kutipan cerita tersebut adalah … pada malam hari.
A. Lakukan sesuatu sesuaikan dengan kemampuan! Perbaikan kalimat (4) pada paragraph tersebut yang tepat adalah
B. Tak perlu membantahorang tua secara berlebihan! ….
C. Ibu harus bertanggung jawab pada anak-anaknya! A. Pelangi akan yampak ketika cuaca mendung atau pada malam
D. Jangan menyerahkan tanggung jawab pada orang lain!
hari,
B. Pelangi tidak akan tampak ketika cuaca cerah, pelangi kelihatan
saat mendung.
C. Oleh karena itu, pelangi akan tampak pada malam hari atau
ketika cuaca cerah.
D. Oleh karena itu, pelangi tidak tampak pada malam hari atau
saat caca mendung.

32. Pulau komodo yang terletak di dekat pulau flores, nusa tenggara
timur menjadi perhatian dunia karena di sana terdapat hewan 34. Pada hari Senin, Kepala MTs Asih Putera akan membuka Masa
langka komodo. Pulau ini bisa dikunjungi kurang lebih 1-2 jam Orientasi Siswa baru bagi kelas VII.
dari pelabuhan terdekat. Populasi komodo di pulau ini mencapai Paragraf pembuka pidato yang tepat adalah …
1300 ekor tahun 2009. Ini sudah lebih dari cukup untuk menarik A. Para siswa sekalian yang berbahagia, Marilah kita memanjatkan
minat wisatawan local maupun asing. Pulau yang luasnya 390 km2 puji syukur kepada Tuhan Yang Maha Esa karena saat ini, kita
ini telah diumumkan sebagai salah satu dari 7 keajaiban dunia. berkumpul dalam acara pembukaan masa orientasi siswa baru.
Perbaikan penuisan ejaan yang tepat pada teks tersebut adalah … B. Para siswa semua yang berbahagia. Marilah kita panjatkan puji
A. Pulau komodo, pulau Flores, Nusa Tenggara Timur. syukur kepada Allah swt. karena saat ini, kita dapat hadir di sini
B. Pulau Komodo, pulau Flores, Nusa Tenggara Timur. dalam rangka pembukaan masa orientasi siswa baru.
C. Pulau Komodo, Pulau Flores, Nusa Tenggara Timur.
C. Para siswa yang berbahagia. Marilah kita panjatkan puji syukur
D.Pulau komodo, pulau Flores, Nusa Tenggara timur.
kehadirat Allah swt. karena berkat izin-Nya, kita dapat hadir di
33. (1) Pelangi dan efek cahaya lain di langit disebabkan oleh cahaya sini dalam rangka pembukaan masa orientasi siswa.
yang membias dan menyimpang menjauhi partikel. (2) Saat D. Para siswa-siswi yang berbahagia. Marilah kita memanjatkan
matahari terbenam, langit menjadi merah karena sinar matahari puji syukur kehadirat Tuhan Yang Maha Esa karena saat ini,
lewat melalui atmosfer yang jauh lebih tebal daripada ketika kita dapat hadir di sini dalam rangka pembukaan masa orientasi
matahari berada tinggi di langit pada siang hari. (3) Pelangi siswa.
35. Campur gula merah dengan air panas! Biarkan hingga dingin dan Cermatilah kutipan cerpen berikut dengan saksama untuk
kemudian tuangkan di separuh bagian potongan bawah botol! mengerjakan soal nomor 38-40!
Petikan di atas merupakan bagian ... teks rekaman percobaan Karyamin melangkah pelan dan sangat hati-hati. Beban yang
A. alat dan bahan C. tujuan menekan pundaknya adalah pikulan yang digantungi dua
B. langkah D. simpulan keranjang batu kali. Jalan tanah yang sedang didakinya sudah licin
dibasahi air yang menetes dari tubuh Karyamin dan kawan-kawan,
36. Kata berimbuhan asing berikut memiliki makna sistem, terdapat yang pulang balik mengangkat batu dari sungai ke pangkalan
pada kalimat … material di atas sana.
A. Amerika adalah penganut kapitalisme sejati. Karyamin sudah berpengalaman agar setiap perjalanannya selamat,
B. Sosialisasi ini dilakukan dalam rangka pengenalan program yakni berjalan menanjak sambil menjaga agar titik berat beban dan
kepada masyarakat. badannya tetap berada pada telapak kaki kiri atau kanannya.
C. Dia terkenal sebagai orang yang aktif berorganisasi. Pemindahan titik berat dari kaki kiri ke kaki kanannya pun harus
D.Amoeba merupakan salah satu organisme bersel satu. dilakukan dengan baik. Karyamin harus memperhitungkan tarikan
napas serta ayunan tangan demi keseimbangan yang sempurna.
(Ahmad Tohari, Senyum Karyamin)
38. Kutipan cerpen tersebut merupakan bagian ...
A. abstrak
37. Cermatilah kutipan cerpen berikut! B. komplikasi
Orang tua itu mendengar apa yang dibicarakan anak dan C. orientasi
menantunya. Dia melihat ke depan, ke arah yang dikatakan anak D. evaluasi
dan menantunya Tampak olehnya patung polisi di bawah guyuran
hujan lebat dalam posisi memberi hormat kepada mereka. Mobil 39. Gambaran dalam teks tersebut menunjukkan sosok tokoh
pun berjalan karena lampu telah hijau. Dari jendela orang tua Karyamin memiliki makna simbol ....
itu melihat ke luar. Dia perhatikan patung polisi itu dalam A. rakyat kecil yang penuh pergulatan dalam kehidupan
guyuran hujan. Dia iba melihat polisi patung itu. Dia tiba- tiba B. pekerjaan masyarakat desa yang begitu berat
tersentak. C. kondisi fisik orang desa yang ringkih
"Ya Allah. Polisi itu … menjadi batu .... " D. pekerja di desa yang sudah berpengalaman
(Hamsad Rangkuti; Si Lugu dan Si Malin Kundang)
Kutipan tersebut merupakan bagian dari struktur cerpen, yaitu .... 40. Keunikan penggunaan bahasa Ahmad Tohari dalam kutipan
A. Abstrak C. Evaluasi cerpen tersebut terlihat dalam ....
B. Resolusi D. Koda A. penggambaran sosok tokoh dan alam sekitar yang begitu detail
B. pemaparan isi hati dan perasaan tokoh menjadi lebih jelas
C. penggambaran alam akan membingungkan pembaca yang tidak
akrab dengan pedesaan 42. Makna kata berkorelasi dalam kutipan teks ulasan tersebut adalah
D. peran penulis sebagai pencerita terlihat sangat jelas ....
A. sering berhubungan secara timbal balik
Cermatilah kutipan ulasan buku berikut untuk mengerjakan soal B. bersama-sama
nomor 41-43! C. saling melengkapi
Rindu adalah persembahan Tere Liye di tahun 2014 yang D. keterkaitan dengan hal-hal nyata
betul-betul dlrindukan. Rindu merupakan buku ke-20 karya
pengarang produktif tersebut. Semua karya-karyanya memiliki ciri 43. Bagian rangkuman dalam teks ulasan buku pada umumnya berisi
khas dan cita rasa yang berbeda. Saya tidak habis pikir, lagi-lagi tentang ....
Tere Liye menyuguhkan tema yang tidak biasa. Menurut saya, ide A. data buku
penulisan novel Rindu belum pernah ada di dunia perbukuan B. latar belakang penulisan
Indonesia. Sederhana, tidak muluk-muluk, tetapi segar. C. kelebihan dan kelemahan buku
Novel ini bercerita tentang perialanan panjang jamaah haji D. manfaat untuk para pembaca
lndoniesia tahun 1938. Tentang kapa uap Blitar Holland, sejarah
Nusantara dan tentang pertanyaan - pertanyaan seputar masa lalu, 44. “Mila, minggu depan kita harus pindah ke Yogyakarta. Ayah
kebencian, takdir, cinta dan kemunafikan. dipindahtugaskan di sana.” Bagaikan petir di siang bolong
Novel ini ditulis dengan alur maju sehingga memudahkan
menyambar Mila, yang seketika itu langsung diam mematung.
pembaca untuk mengikuti jalan cerita. Namun, di beberapa
bagian, penulis menyuguhkan cerita-cerita lain dalam bentuk Makna symbol petir di siang bolong dalam kutipan ceirta tersebut
dialog, yang berkorelasi pada kisah yang tengah disajikan. Hal adalah …
tersebut rnembuat pembaca mengenal secara racikan cerita di novel
A. Merasa sedih
ini sehingga setting novel yang didominasi aktivitas penumpang di
kapal Holland tidak terasa membosankan. B. Merasa bingung
Gaya kepenulisan novel Rindu terbilang sederhana dan C. Sangat terkejut
membumi. Disisipi dialog bahasa Belanda, yang meski tidak
D. Sangat heran
disertakan artinya, pembaca terbantu rnernahami maksud
kalimat dengan deskripsi yang ditulis Tere Liye.
Bacalah kutipan drama berikut untuk menjawab soal nomor 45-46!
41. Paragraf ketiga teks bagian ulasan buku tersebut merupakan .... (DI RUANG TENGAH TERJADI PERDEBATAN SERU)
A. orientasi C. evaluasi Faiz : Mengapa begitu banyak kata harus, jangan, wajib? Saya tidak
B. tafsiran D. rangkuman sanggup memenuhinya. Jangan paksa saya! Saya tidak berbakat
menjadi orang berdarah dingin. Saya tidak berani menjadi kejam, Cermatilah kutipan teks laporan berikut untuk mengerjakan
penuh tipu daya. Saya ingin jadi saya sendiri. soal nomor 48-49!
Mario : Maksudnya? Puluhan ribu para transmigran yang tinggal di berbagai wilayah
Faiz : Ayah adalah ayah. Dengan bakat besar untuk menciptakan di Kalimantan Selatan berbeda-beda nasibnya. Ada yang sukses,
usaha, mencetak uang. Saya adalah saya, bodoh, dan dungu, bahkan tetapi tidak sedikit yang gagal. Di antara mereka yang gagal, ada
menghitung recehan pun tak mampu. yang tetap bersabar sambil berusaha lebih keras memperbaiki
Mario : Bukan, Bukan itu. Apa maksudnya berdarah dingin, kejam, kehidupan mereka. Bagi yang tidak mau bersabar, banyak yang
penuh tipu daya? Ke mana arah pembicaraanmu itu? kembali ke Jawa walaupun mereka tidak memiliki apa-apa lagi di
Faiz : Saya tidak ingin membeberkan apa yang saya tahu. daerah asalnya.

45. Latar tempat dalam kutipan drama di atas adalah … 48. Kesalahan penggunaan kata dalam kalimat pertama teks laporan
A. ruang tamu tersebut adalah ....
B. ruang tengah A. kata "berbeda-beda" seharusnya ditulis "berbeda"
C. dapur
B. kata “para” yang mubazir
D. depan rumah
C. kata “puluhan ribu” seharusnya dihilangkan
46. Watak tokoh Faiz dalam penggalan drama di atas adalah … D. kata “tetapi” seharusnya diganti “namun”
A. tegas C. pemarah
B. pembangkang D. pemalu 49. Kesalahan ejaan pada kalimat terakhir teks laporan tersebut
adalah ....
47. (1) Hampir setiap malam mereka berkumpul bersama, berpesta, A. kata "kembali” seharusnya diganti “pulang”
menari, dan bergembira. Mereka saling berbagi makanan kecuali B. kata "walaupun" seharusnya diganti "meskipun"
seekor belalang yang selalu hidup menyendiri. Ia hanya memandang C. kata "kembali" seharusnya diganti ''dikembalikan"
keramaian dari depan rumahnya. (2) Tingkah belalang itu sangat D. kata "asalnya" seharusnya diganti "asal mereka"
aneh, ia malu karena ia telah kehilangan sebuah kakinya. (3) Kakek
Cacing pernah bercerita, Paman Belalang setahun yang lalu telah
50. Pakar transportasi […] bahwa [emerinth lebih baik membangun
kehilangan kakinya akibat ia berkelahi dengan seekor burung yang
infrastruktur transportasi laut dan udara karena Indonesia
hendak memangsanya. (4) Sehari-hari Paman Belalang hanya duduk
merupakan Negara kepulauan.
termenung meratapi kakinya yang hilang. Kata bentukan yang tepat untuk melengkapi
Makna penyesalan dalam fabel di atas terdapat pada kalimat nomor A. menunjukkan C.meningkatkan
… B. menanyakan D.menyarankan
A. (1) B. (2) C. (3) D. (4)
***
Man Jadda Wa Jadda
“Barang siapa yang
bersungguh-sungguh pasti akan
berhasil.”

Tidak ada hal yang sulit jika kita


mau berusaha, di mana ada
kemauan di situ pasti ada jalan

uma_file@101018

Anda mungkin juga menyukai